High Yield Surgery

You might also like

Download as pdf or txt
Download as pdf or txt
You are on page 1of 101

High Yield Surgery

Shelf Exam Review


Emma Holliday Ramahi
Pre-Op Evaluation
• Contraindications to surgery
– Absolute? Diabetic Coma, DKA

– Poor nutrition? albumin <3, transferrin <200,


weight loss <20%.
– Severe liver failure? bili >2, PT >16, ammonia > 150
or encephalopathy
– Smoker? stop smoking 8wks prior to surgery

If a CO2 retainer, go easy on the O2 in the post-op


period. Can suppress respiratory drive.
Tells you who is at
• Goldman’s Index  greatest risk for surgery
– #1 = CHF
• What should you check? EF. If <35%, no surg.
– #2 = MI w/in 6mo
EKG  stress test 
• What should you check? cardiac cath  revasc.
– #3 = arrhythmia
– #4 = Old (age >70)
– #5 = Surgery is emergent
– #6 = AS, poor medical condition, surg in chest/abd
• What should you check?
Listen for murmur of AS-
Late systolic, crescendo-decrescendo murmur that radiates
to carotids. ↑ with squatting, ↓ with decr preload
Aspirin, NSAIDs, vit E (2wks)
• Meds to stop:
Warfarin (5 days) – drop INR to
<1.5 (can use vit K)
Take ½ the morning dose of
insulin, if diabetic
• If CKD on dialysis: Dialyze 24 hours pre-op
• Why do we check the BUN and Creatinine?
– What is the worry if BUN > 100?
There is an increased risk of post-op bleeding 2/2
uremic platelet dysfunction.
– What would you expect on coag pannel?
Normal platelets but prolonged bleeding time
Vent Settings
set TV and rate but if pt takes a
• Assist-control  breath, vent gives the volume.

• Pressure support  pt rules rate but a boost of


*Important for weaning.* pressure is given (8-20).

• CPAP  pt must breathe on own but + pressure


given all the time.

• PEEP  pressure given at the end of


*Used in ARDS or CHF* cycle to keep alveoli open
(5-20).
You have a patient on a vent…
• Best test to evaluate management? ABG
• If PaO2 is low? increase FiO2
• If PaO2 is high? decrease FiO2
• If PaCO2 is low (pH is high)?  Decr rate or TV
• If PaCO2 is high (pH is low)? Incr rate or TV
• Which is more efficient? TV is more efficient to
change.
*Remember minute
ventilation equation
& dead space*
Acid Base Disorders
• Check pH  if <7.4 = acidotic.
• Next  Check HCO3 and pCO2:
– If HCO2 is high and pCO2 is high? Respiratory Acidosis
– If HCO2 is low and pCO2 is low? Metabolic Acidosis
• Next  Check anion gap (Na – [Cl + HCO3]), normal? 8-12
• Gap acidosis = MUDPILES
• Non-gap acidosis = diarrhea, diuretic, RTAs (I< II, IV)
• Check pH  if >7.4 = alkalotic.
• Next  Check HCO3 and pCO2:
– If HCO3 is low and pCO2 is low  Respiratory Alkalosis
– If HCO3 is high and pCO2 is high  Metabolic Alkalosis
• Next  Check urine [Cl]
• If [Cl] < 20 Vomiting/NG,
antactids, diuretics
• If [Cl] > 20
Conn’s, Bartter’s Gittleman’s.
Sodium Abnormalities
• ↓Na = Gain of water
– Check osm, then check volume status.
– ↑volume ↓Na: CHF, nephrotic, cirrotic
– ↑volume ↓ Na: diuretics or vomiting + free water
– Nl volume ↓Na: SIADH, Addisons, hypothyroidism.
– Treatment? Fluid restriction & diruetics
– If hypovolemic? Normal Saline
– When to use 3% saline? Symptomatic (Seizures), < 110
– What would you worry about? Central Pontine Myolinolysis.
• ↑Na = Loss of water
– Treatment? Replace w/ D5W or hypotonic fluid
– What would you worry about? cerebral
edema.
Other Electrolyte Abnormalities
• Numbness, Chvostek or Troussaeu, prolonged
QT interval. ↓Ca
• Bones, stones, groans, psycho. Shortened QT
interval. ↑Ca
• Paralysis, ileus, ST depression, U waves.↓K
– Treatment? give K (kidneys!), max 40mEq/hr
• Peaked T waves, prolonged PR and QRS, sine
waves. ↑K
– Treatment? Give Ca-gluconate then insulin + glc,
kayexalate, albuterol and sodium
bicarb. Last resort = dialysis
Fluid and Nutrition
• Maintenance IVFs  D51/2NS + 20KCl (if peeing)
– Up to 10kg s 100mL/kg/day
– Next 10 kgs  50mL/kg/day
– All above 20  20mL/kg/day
• Enteral Feeds are best  keep gut mucosa in tact
and prevent bacterial translocation.
• TPN is indicated if gut can’t absorb nutrients 2/2
physical or fxnal loss.
– Risks = *acalculus cholecystitis*, hyperglycemia, liver
dysfxn, *zinc deficiency*, other ‘lyte probs
Burn

www.readykor.com/docs/burns_files/burns9.jpg

http://emedicine.medscape.com/arti
http://en.wikipedia.org/wiki cle/769193-media
1st degree /Burn

2nd degree 3rd degree

• Circumferential burns? Consider escharotomy


• Look for singed nose hairs, wheezing, soot in
mouth/nose? Low threshold for intubation
• Patient w/ confusion, HA, cherry red skin?
– Best test? Check carboxyHb (pulse ox = worthless)
– Treatment? 100% O2 (hyperbaric if CO-Hb is ↑↑↑
Clotting & Bleeding
• Clotting-
– In old people? Think cancer
– Edema, HTN, & foamy pee? Nephrotic syndrome
– In young person w/ +FH Factor V Leiden
– What’s special about ATIII def? Heparin won’t work
– Young woman w/ mult. SABs? Lupus Anticoagulant
– Post op, ↓plts, clots HIT! (If heparin w/in 5-14 days
• What do you treat w/?
Leparudin or agatroban
• Bleeding
– Isolated decr in plts? ITP
– Normal plts but incr bleeding time & PTT? vWD
– Low plts, Incr PT, PTT, BT, low fibrinogen, high Ddimer
and schistocytes? DIC!! Caused by gram – sepsis,
carcinomatosis, OB stuff
Burn Work up and Tx
• Rule of 9s – Parkland formula-
• Adults- Kg x % BSA x 3-4
Give ½ over the •
1st 8hrs and the • Kiddos- Kg x % BSA x 2-4
rest over next •
• Ringers lactate or
16hrs
normal saline
• NO PO or IV abx. Give topical.
http://img.tfd.com/dorland/thumbs/rule_of-nines.jpg

Silver
• Doesn’t penetrate eschar and can cause
Sulfadiazine
leukopenia?
• Penetrates eschar but hurts like hell? Mafenide
• Doesn’t penetrate eschar and causes hypoK and
HypoNa? Silver Nitrate
Other Burn Stuff
• Chemical burn, what to do? Irrigate >30min prior to ER
• Electrical Burn, best 1st step? EKG!
• If abnormal? 48 hours of telemetry (also if LOC)
• If urine dipstick + for blood but microscopic exam
is negative for RBCs? Myoglobinuria  ATN
• Then what do you check? K+! (When cells break)
• If affected extremity is extremley tender, numb,
white, cold with barely dopplerable pulses?
Compartment syndrome!!
– Criteria? 5 Ps or compartment pressure >30mmHg
– Treatment? May require fasciotomy. (at bedside!)
Trauma Drama
• Airway-
– If trauma patient comes in unconscious? Intubate!
– If GCS < 8? Intubate!
– If guy stung by a bee, developing stridor and
tripod posturing? Intubate!
– If guy stabbed in the neck, GCS = 15, expanding
mass in lateral neck? Intubate!
– If guy stabbed in the neck, crackly sounds w/
palpating anterior neck tissues? fiberoptic
broncoscope
– If huge facial trauma, blood obscures oral and
nasal airway, & GCS of 7? cricothyroidotomy
• Breathing-
– So you intubated your patient… next best step?
Check bilateral breath sounds
– If decr on the left?
Means you intubated the right mainstem bronchus
– What to do? Pull back your ET tube
– Next step? Check pulse ox, keep it >90%
www.imagingpathways.health.wa.gov.au/.../cxr.jpg www.daviddarling.info/images/pneumothorax_rad...

Traumatic Aortic Injury Pneumothorax


img.medscape.com/.../424545-425518-718tn.jpg
upload.wikimedia.org/.../Pulmonary_contusion.jpg

Hemothorax Pulmonary Contusion


Chest Trauma
• A patient has inward mvmt of the right ribcage
upon inspiration.
– Dx? Flail chest. >3 consec rib fractures
– Tx? O2 and pain control. With what?*
• A patient has confusion, petechial rash in chest,
axilla and neck and acute SOB.
– Dx? Fat embolism
– When to suspect it? After long bone fx (esp femur)
• A patient dies suddenly after a 3rd year medical
student removes a central line.
– Dx? Air embolism
– When else to suspect it? Lung trauma, vent use, during
heart vessel surgery.
• Cardiovascular-
Worry about shock
– If hypotensive, tachycardic?
Hypovolemic/
– If flat neck veins and normal CVP? Hemorrhagic
– Next best step? 2 large bore periph IV- 2L NS or LR over
20min followed by blood.
– If muffled <3 sounds, JVD, electrical alternans,
pulsus paradoxus? Pericardial Tamponade
• Confirmatory test? FAST scan
• Treatment? Needle decompression, pericardial window or
median sternotomy
– If decr BS on one side, tracheal deviation AWAY
from collapsed lung? Tension Pneumothorax
• Next best step? Needle decompression, followed by
a chest tube.
DON’T do a CXR!!!
Shock
Types of Shock Causes Physical Exam Swan-Ganz Treatment
Catheter
Hypovolemic Loss of circulating blood volume (whole Hypotensive, tachycardic, RAP/ PCWP↓ Crystalloid
blood from hemorrhage or interstitial from diaphoretic, cool, clammy resuscitation
SVR↑
bowel obstruction, excessive vomiting or extremities
diarrhea, polyuria or burn) CO↓
Vasogenic Decreased resistance w/in capacitance Altered mental status, RAP/PCWP↓ Fluid resuscitation
vessels, seen in sepsis (LPS) and hypotension warm, dry (may cause edema)
SVR↓
anaphylaxis (histamine) extremities (early), Late and tx offending
looks like hypovolemic CO↑ (EF↓) organism
Neurogenic A form of vasogenic shock where spinal Hypotensive, bradycardic, RAP/PCWP↓ In adrenal insuff, tx
cord injury, spinal anesthesia, or adrenal warm, dry extremities, w/ dexamethasone
SVR↓
insufficiency (suspect in pts on steroids absent reflexes and flaccid and taper over
encountering a stressor) causes an acute tone. Adrenal insuf will CO↑ several weeks.
loss of sympathetic vascular tone have hypoNa, hyperK
Cardio- Cardiac tamponade or other processes Hypotensive, tachycardic, U/S shows fluid Pericardio-centesis
compressive exerting pressure on the heart so it cannot JVD, decreased heart in the pericardial performed by
fulfill its role as a pump sounds, normal breath space inserting needle to
sounds, pulsus paradoxus pericardial space
Cardiogenic Failure of the heart as a pump, as in SOB, clammy extremities, RAP/PCWP↑ give diuretics up
arrhythmias or acute heart failure rales bilaterially, S3, front, tx the HR to
SVR↑
pleural effusion, decr 60-100, then address
breath sounds, ascites, CO↓ rhythm. Next give
periph edema, vasopressor support
if nec.
Head Trauma
• GCS  eyes 4, motor 6, verbal 5

tumj.tums.ac.ir
uiowa.edu
prep4usmle.com

Epidural Acute subdural Chronic subdural


Hematoma, edema, tumor can cause increased ICP
Symptoms? Headache, vomiting, altered mental status
Elevate HOB, hyperventillate to pCO2 28-32,
Treatment?
give mannitol (watch renal fxn)
Surgical intervention?
Ventriculostomy
Neck Trauma
Penetrating Trauma  GSW
or stab wound
Zone 3 = ↑ angle of mandible
w/u? Aortography and triple
endoscopy.
Zone 2 = angle of mandible-cricoid
w/u? 2D doppler +/- exploratory
surgery.
Zone 1 = ↓ cricoid
w/u? Aortography
Penetrating Abdominal Trauma
• If GSW to the abdomen?
Ex-lap. (plus tetanus prophylaxis)
• If stab wound & pt is unstable,
with rebound tenderness &
rigidity, or w/ evisceration?
Ex-lap. (plus tetanus prophylaxis)

If you see this? • If stab wound but pt is stable?


FAST exam. DPL if FAST is equivocal.
Do not pass go, go Ex-lap if either are positive.
directly to • If blunt abdominal trauma pt
exploratory with hypotension/tachycardia:
laparotomy. Ex-lap.
Blunt Abdominal Trauma
If unstable? Ex-lap.
If stable? Abdominal CT
Spleen or
– If lower rib fx plus bleeding into abdomen liver lac.
– If lower rib fx plus hematuria Kidney lac.
– If Kehr sign & viscera in thorax on CXR Diaphragm
rupture.
– If handlebar sign Pancreatic rupture.
– If stable w/ epigastric pain?
• Best test? Abdominal CT.
• If retroperitoneal fluid is found? Consider duodenal
rupture.
Pelvic Trauma
FAST and DPL to r/o bleeding in
• If hypotensive, tachycardic  abdominal cavity.
• Can bleed out into pelvis  stop bleeding by fixing fx 
internal if stable, external if not.
• If blood at the urethral meatus and a high riding prostate?
Consider pelvic fracture w/ urethral or bladder injury.
• Next best test? Retrograde urethrogram (NOT FOLEY!)
• If normal? Retrograde cystogram to evaluate bladder
• What are you looking for? Check for extravasation of dye. Take
2 views to ID trigone injury.
If extraperitoneal extravasation?
Bed rest + foley
If intraperitoneal extravasation?
Ex-lap and surgical repair
Ortho Trauma
• Fractures that go to the OR-
– Depressed skull fx
– Severely displaced or angulated fx
– Any open fx (sticking out bone needs cleaning)
– Femoral neck or intertrochanteric fx

• Common fractures-
– Shoulder pain s/p seizure or electrical shock Post. shoulder dislocation
– Arm outwardly rotated, & numbness over deltoid. Ant. shoulder dislocation
– old lady FOOSH, distal radius displaced. Colle’s fracture
– young person FOOSH, anatomic snuff box tender. Scaphoid fracture
– “I swear I just punched a wall…” Metacarpal neck fracture “Boxer’s
fracture”. May need K wire
– Clavicle most commonly broken where? Between middle and distal 1/3s.
Need figure of 8 device
Ortho Trauma X-rays

Depressed skull fx Colle’s fx Scaphoid fx


mksforum.net xraypedia.com/files/images/fxapcolles.jpg
orthoinfo.aaos.org/figures/A00012F04.jpg

Clavicle fx Femoral neck fx


en.academic.ru
gentili.net Intertrochanteric fx
download.imaging.consult.com/.../gr5-
midi.jpg
• Fever on POD #1-
– Most common cause, low fever (<101) and non
productive cough? Atalectasis
• Dx? CXR- see bilateral lower lobe fluffy infiltrates
• Tx? Mobilization and incentive spirometry.
– High fever (to 104!!), very ill appearing. Nec Fasc
• Pattern of spread? In subQ along Scarpa’s fascia.
• Common bugs? GABHS or clostridium perfringens
• Tx? IV PCN, Go to OR and debride skin until it bleeds
– High fever (>104!!) muscle rigidity. Malignant
• Caused by? Succ or Halothane Hyperthermia
• Genetic defect? Ryanodine receptor gene defect
• Treatment? Dantrolene Na (blockes RYR and decr
intracellular calcium.
• Fever on POD #3-5-
– Fever, productive cough, diaphoresis
Pneumonia acutemed.co.uk

• Tx? Check sputum sample for culture, cover w/ moxi


etc to cover strep pneumo in the mean time.
– Fever, dysuria, frequency, urgency, particularly in a
patient w/ a foley.
UTI
• Next best test? UA (nitritie and LE) and culture.
• Tx? Change foley and treat w/ wide-spec abx until
culture returns.
• Fever > POD 7-
Central line infection
– Pain & tenderness at IV site
• Tx? Do blood cx from the line. Pull it. Abx to cover staph.
– Pain @ incision site, edema, induration Cellulits
but no drainage.
• Tx? Do blood cx and start antibiotics Simple
– Pain @ incision site, induration WITH drainage.Wound
• Tx? Open wound and repack. No abx necessary Infection
– Pain w/ salmon colored fluid from incision. Dehiscence
• Tx? Surgical emergency! Go to OR, IV abx, primary closure of fascia
– Unexplained fever Abdominal Abscess
• Dx? CT w/ oral, IV and rectal contrast to find it. Diagnostic lap.
• Tx? Drain it! Percutaneously, IR-guided, or surgically.
– Random  thyrotoxicosis, thrombophlebitis, adrenal
insufficiency, lymphangitis, sepsis.
Pressure Ulcers
• Caused by impaired blood flow  ischemia
– Don’t culture  will just get skin flora. Check CBC and blood cultures.
Can mean bacteremia or osteomyelitis.
– Can do tissue biopsy to rule out Marjolin’s ulcer
– Best prevention is turning q2hrs

– Stage 1 = skin intact but red. Blanches w/ pressure judy-


waterlow.co.
uk

– Stage 2 = blister or break in the dermis


qondio.com

– Stage 3 = SubQ destruction into the muscle

– Stage 4 = involvement of joint or bone. gndmoh.com seejanenurse.wordpress.com

• Stage 1-2 get special mattress, barrier protection


• Stage 3-4 get flap reconstruction surgery
– Before surgery, albumen must be >3.5 and bacterial load must be
<100K
Thoracic
• Pleural Effusions  see fluid >1cm on lat decu
 thoracentesis!
– If transudative, likely CHF, nephrotic, cirrhotic
• If low pleural glucose? Rheumatoid Arthritis
• If high lymphocytes? Tuburculosis
• If bloody? Malignant or Pulmonary Embolus
– If exudative, likely parapneumonic, cancer, etc.
– If complicated (+ gram or cx, pH < 7.2, glc < 60):
• Insert chest tube for drainage.
– Light’s Criteria  transudative if:
LDH < 200
LDH eff/serum < 0.6
Protein eff/serum < 0.5
ncbi.nlm.nih.gov
• Spontaneous Pneumothorax  subpleural bleb
ruptures  lung collapse.
– Suspect in tall, thin young men w/ sudden dyspnea (or
asthma or COPD-emphysema)
– Dx w/ CXR, Tx w/ chest tube placement
– Indications for surgery = ipsi or contra recurrence,
bilateral, incomplete lung expansion, pilot, scuba, live
in remote area  VATS, pleurodesis (bleo, iodine or
talc)
• Lung Abscess  usually 2/2 aspiration (drunk,
elderly, enteral feeds)
– Most often in post upper or sup lower lobes
– Tx initially w/ abx  IV PCN or clinda
– Indications for surgery = abx fail,
abscess >6cm, or if empyema is present.

www.meddean.luc.edu
Work up of a Solitary Lung Nodule
• 1st step = Find an old CXR to compare!
• Characteristics of benign nodules:
– Popcorn calcification = hamartoma (most common)
– Concentric calcification = old granuloma
– Pt < 40, <3cm, well circumscribed
– Tx? CXR or CT scans q2mo to look for growth
• Characteristics of malignant nodules:
http://emedicine.medscape.com/
article/356271-media

– If pt has risk factors (smoker, old), If >3cm, if eccentric


calcification
– Tx? Remove the nodule (w/ bronc if central,
open lung biopsy if peripheral. http://emedicine.medscape.com/ar
ticle/358433-media
A patient presents with weight loss, cough,
dyspnea, hemoptysis, repeated pnia or lung
collapse.
• MC cancer in non-smokers? Adenocarcinoma. Occurs in scars of old pnia
• Location and mets? Peripheral cancer. Mets to liver, bone, brain and adrenals
• Characteristics of effusion? Exudative with high hyaluronidase
• Patient with kidney stones, Squamous cell carcinoma.
constipation and malaise low PTH +Paraneoplastic syndrome 2/2 secretion
central lung mass? of PTH-rP. Low PO4, High Ca
• Patient with shoulder pain, ptosis, Superior Sulcus Syndrome from Small
constricted pupil, and facial edema? cell carcinoma. Also a central cancer.
• Patient with ptosis better after 1 Lambert Eaton Syndrome from small
minute of upward gaze? cell carcinoma. Ab to pre-syn Ca chan
• Old smoker presenting w/ Na = 125, SIADH from small cell carcinoma.
moist mucus membranes, no JVD? Produces Euvolemic hyponatremia.
• CXR showing peripheral cavitation andFluid restrict +/- 3% saline in <112
CT showing distant mets? Large Cell Carcinoma
ARDS
• Pathophys: inflammation  impaired www.ispub.com/.../ards3_thumbnail.gif

gas xchange, inflam mediator release,


hypoxemia
• Causes:
– Sepsis, gastric aspiration, trauma, low perfusion,
pancreatitis.
• Diagnosis: 1.) PaO2/FiO2 < 200 (<300 means acute lung injury)
2.) Bilateral alveolar infiltrates on CXR
3.) PCWP is <18 (means pulmonary edema is non-cardio

• Treatment: Mechanical ventilation w/ PEEP


Murmur Buzzwords

• SEM cresc/decresc, louder w/


Aortic Stenosis
squatting, softer w/ valsalva. +
parvus et tardus
• SEM louder w/ valsalva, softer HOCM
w/ squatting or handgrip.
• Late systolic murmur w/ click Mitral Valve Prolapse
louder w/ valsalva and
handgrip, softer w/ squatting
• Holosystolic murmur radiates Mitral Regurgitation
to axilla w/ LAE
More Murmurs

• Holosystolic murmur w/ late


VSD
diastolic rumble in kiddos
• Continuous machine like PDA
murmur-
• Wide fixed and split S2- ASD

• Rumbling diastolic murmur Mitral Stenosis


with an opening snap, LAE and
A-fib
• Blowing diastolic murmur with Aortic Regurgitation
widened pulse pressure and
eponym parade.
• Bad breath & snacks in Zenker’s diverticulum.
the AM. Tx w/ surgery

• True or false? False. Only contains mucosa


• Dysphagia to liquids & solids. Dysphagia worse w/ hot &
Achalasia. cold liquids + chest pain that
Tx w/ CCB, nitrates, feels like MI w/ NO regurg
botox, or heller
sxs. Diffuse esphogeal spasm.
myotomy
Assoc w/ Chagas dz Tx w/ CCB or nitrates
jykang.co.uk
and esophageal
cancer.
ajronline.org

• Epigastric pain worse after GERD. Most sensitive test is 24-hr pH


eating or when laying down monitoring. Do endoscopy ifst“danger signs”
present. Tx w/ behav mod 1 , then antacids,
cough, wheeze, hoarse. H2 block, PPI.
• Indications for surgery? bleeding, stricture, Barrett’s, incompetent LES,
max dose PPI w/ still sxs, or no want meds.
If hematemesis (blood occurs If gross hematemesis If progressive
after vomiting, w/ subQ unprovoked in a cirrhotic dysphagia/wgt loss.
emphysema). Can see pleural w/ pHTN. Esophageal Carcinoma
effusion w/ ↑amylase Gastric Varices Squamous cell in
Boerhaave’s If in hypovolemic shock? smoker/drinkers in the
Esophageal Rupture middle 1/3.
do ABCs, NG lavage, Adeno in ppl with long
Next best test? medical tx w/ octreotide standing GERD in the
CXR, gastrograffin or SS. Balloon distal 1/3.
esophagram. NO tamponade only if you
edoscopy need to stablize for Best 1st test?
Tx? transport
barium swallow, then
surgical repair if full Tx of choice? endoscopy w/ bx, then
thickness staging CT.
Endoscopic
sclerotherapy or
banding
*Don’t prophylactically
band asymptomatic
varices. Give BB. img.medscape.com
/pi/emed/ckb/onco
logy/276262
Stomach
http://emedicine.medscape.c
om/article/175765-media

• Acid reflux pain after eating, when laying down- Hiatal Hernia
– Type 1 = Sliding. GE jxn herniates into thorax. Worse for GERD. Tx sxs.
– Type 2 =Paraesophageal. Abd pain, obstruction, strangulation  needs surgery.
• MEG pain worse w/ eating. H.pylori, NSAIDs, ‘roids- Gastric Ulcers
Double-contrast barium swallow- punched out lesion w/ reg margins.
– Work up =
EGD w/ bx can tell H. pylori, malign, benign.
– Surgery if-
Lesion persists after 12wks of treatment.
• Gastric Cancer- Adeno most common. Esp in Japan
– Krukenberg Gastric CA  ovaries Blummer’s Shelf Mets felt on DRE
Virchow’s node L supraclav fossa Sister Mary Joseph Umbilical node
– Lymphoma- HIV MALT-lymphoma- H. pylori
• Randoms-
– Mentriers = protein losing enteropathy, enlarged rugae.
– Gastric Varices = splenic vein thrombosis.
– Dieulafoy’s = massive hematemesis  mucosal artery erodes into
stomach
Duodenum
• MEG pain better w/ eating Duodenal Ulcers
– 95% assoc w/ H. pylori
– Healthy pts < 45y/o can do trial of H2 block or PPI
– Dx? blood, stool or breath test for H. pylori but endoscopy w/
biopsy (CLO test) is best b/c it can also exclude cancer.
– Tx? PPI, clarithromycin & amoxicillin for 2wks. Breath or stool
test can be test of cure.
• What to suspect if MEG pain/ulcers don’t resolve? ZE Syndrome
– Best test? Secretin Stim Test (find inapprop high gastrin)
– Tx? Surgical resection of pancreatic/duodenal tumor
– What else to look for? Pituitary and Parathyroid problems.
• A patient has bilious vomiting and post-prandial pain.
Recently lost 200lbs on “Biggest Loser”. SMA Syndrome
– Pathophys- 3rd part of duodenum compressed by AA and SMA
– Tx? by restoring weight/nutrition. Can do Roux-en-Y
Exocrine Pancreas
• MEG pain straight through to the back. Pancreatitis
– Most common etiologies? Gallstones & ETOH
– Dx? Incr amylase & lipase. CT is best imaging test
– Tx? NG suction, NPO, IV rehydration and observation
– Bad prognostic factors- old, WBC>16K, Glc>200, LDH>350, AST>250…
drop in HCT, decr calcium, acidosis, hypox
– Complications- pseudocyst (no cells!), hemorrhage, abscess, ARDs
• Chronic Pancreatitis-
– Chronic MEG pain, DM, malabsorption (steatorrhea)
– Can cause splenic vein thrombosis  which leads to …? Gastric varices!
• Adenocarcinoma-
– Usually don’t have sxs until advanced. If in head of pancreas 
Courvoisier’s sign large, nontender GB, itching and jaundice
– Trousseau’s sign = migratory thrombophlebitis
– Dx w/ EUS and FNA biopsy
– Tx w/ Whipple if: no mets outside abdomen, no extension into SMA or
portal vein, no liver mets, no peritoineal mets.
Endocrine Pancreas
• Insulinoma- sxs (sweat, tremors, hunger, seizures) + BGL <
– Whipple’s triad? 45 + sxs resolve w/ glc admin
– Labs? insulin ↑, C-peptide ↑, pro-insulin ↑
• Glucagonoma-
– Sxs? Hyperglycemia, diarrhea, weight-loss
– Characteristic rash? necrolytic migratory erythema
• Somatistainoma- img.medscape.com/.../104

– Commonly malignant. see malabsorption, 8885-1093550-244.jpg

steatorrhea, ect from exocrine pancreas malfxn


• VIPoma-
– Sxs? Watery diarrhea, hypokalemia, dehydration, flushing.
– Looks similar to carcinoid syndrome.
– Tx? Octreotide can help sxs
Gallbladder Acute Cholecystitis
• RUQ pain  back, n/v, fever, worse s/p fatty foods.
– Best 1st test? U/S
– Tx? Cholecystectomy. Perc cholecystostomy if unstable med-ed.virginia.edu

• RUQ pain, high bili and alk-phos. Choledocolithiasis


– Dx? U/S will show CBD stone.
– Tx? Chole +/- ERCP to remove stone
• RUQ pain, fever, jaundice, ↓BP, AMS. Ascending Cholangitis
– Tx? w/ fluids & broad spec abx. ERCP and stone removal.
• Choledochal cysts-
– Type 1? Fusiform dilation of CBD  Tx w/ excision
– Type 5? Caroli’s Dz. Cysts in intrahepatic ducts  needs liver transplant
• Cholangiocarcinoma- rare.
– Risk factors? Primary sclerosing cholangitis (UC), liver flukes and
thorothrast exposure. Tx w/ surgery +/- radiation.
Liver
• Hepatitis-
– AST = 2x ALT  Alcoholic heptatitis (reversible)
– AST > ALT high (1000s)  Viral hepatitis
– AST & ALT high s/p hemorrhage, surg, or sepsis  Shock liver
• Cirrhosis and Portal HTN-
– Tx- SS and VP vasocontrict to decrease portal pressure, betablockers
also decrease portal pressure.
– Don’t need to treat esophageal varices prophyactically, but
band/burn them once they bleed once.
– TIPS relieves portal HTN but…  worsens hepatic encephalopahty
• Treat with: Lactulose. helps rid body of ammonia.
• Hepatocellular Carcinoma
– RF- chronic hepB carrier > hepC. Cirrhosis for any
reason, plus aflatoxin or carbon tetrachloride.
– Dx w/ high AFP (in 70%), CT/MRI.
– Tx: can surgically remove solitary mass, use rads or cryoablation for
pallation of multiple.
More Liver
*Women on OCP  palpable abd mass or spontaneous
rupture  hemorrhagic shockHepatic Adenoma www.radswiki.net/main/images/thumb/
3/3e/Hepat...

Dx? U/S or MRI


Tx? D/c OCPs. Resect if large or pregnancy is desired
*2nd MC benign liver tumor. W>M but less likely to rupture. Focal Nodular
No tx needed. Hyperplasia
*Bacterial Abscess.
Most common bugs? E. coli, bacteriodes, enterococcus.
Tx? Surgical drainage and IV abx.
RUQ pain, profouse sweating and rigors, palpable liver. Entamoeba histolytica
Tx? Metronidazole. DON’T drain it.
Patient from Mexico presents w/ RUQ and large liver cysts found
on U/S Enchinococcus.
– Mode of transmission? Hydatic cyst paracyte from dog feces.
– Lab findings? eosinophilia, +Casoni skin test
– Tx? albendazole and surgery to remove ENTIRE cyst,
rupture  anaphylaxis
Spleen
• Post-Splenectomy 
– Post op thrombocytosis >1mil  give aspirin. img.medscape.com/.../432648-432823-
3042.jpg

– Prophylactic PCN + S. pneumo, H. flu and N. meningitidis vaccines.


• ITP-
– Consider in isolated thrombocytopenia (bleeding gums, petechiae,
nosebleeds).
– Decr plt count, incr megakaryocytes in marrow.
– NO splenomegaly.
– Tx w/ steroids 1st. If relapse  splenectomy.
• Hereditary Spherocytosis-
– See sxs of hemolytic anemia (jaundice, incr indir bili, LDH, decr
haptoglobin, elevated retic count) + spherocytes on smear and
+osmotic frag test. Prone to gallstones.
– Tx w/ splenectomy (accessory spleen too).
• Traumatic Splenic Rupture- www.ezhemeonc.com/wp-
content/upload

– Consider w/ L lower rib fx and intra abd hemorrhage. Can have Kehr’s
sign (irritates L diaphragm).
Appendix
• pain in umbilical area  RLQ, n/v.
perf. Appendicitis
– Go to surgery if: Clinical picture is convincing.
– If perforated/abscess? drain, abx (to cover e.coli & bacteriodes),
and do interval appendectomy
• Carcinoid Tumor- #1 site: Appendix!
– Carcinoid syndrome sxs? Diarrhea, Wheezing.
– When do they happen? When mets to liver. (1st pass metabolism)
– What else to look out for? Diarrhea, Dementia, Dermatitis
– If >2cm, @ base of appendix, or
w/ + nodes  Hemicolectomy
– Otherwise  Appendectomy is good enough
Bowel Obstruction
• Small Bowel Obstruction-
– Suspect in hernia, prior GI surgery (adhesions), cancer,
intussusception, IBD.
– Sxs are pain, constipation, obstipation, vomiting.
– 1st test is upright CXR to look for free air. CT can show point of
obstruction.
– Tx w/ IVF, NG tube. Do surgery if peritoneal signs, Incr WBC, no
improvement w/in 48hrs.
• Volvulus- either cecal or sigmoid
– Decompression from below if not strangulated. Otherwise, need
surgical removal and colostomy.
• Post-Op Ileus-
– Also consider if hypoK (make sure to replete), opiates.
– See dilated loops of small bowel w/ air-fluid level.
– Do surgery for perforation. Give lactulose/erythromycin.
• Ogilvie’s syndrome-
– See massive colonic distension. If >10cm, need decompression w/ NG
tube and neostigmine (watch for bradycardia) or colonoscopic
decompression.
Abdominal Imaging

http://emedicine.medscape.com/article/ http://www.ganfyd.org/index.php?tit http://emedicine.medscape.com/ar


374962-overview le=Small_bowel_obstruction ticle/178948-media

http://emedicine.medscape.co
http://emedicine.medscape.com/article/7 m/article/774045-
74045-diagnlearningradiology.comosis diagnlearningradiology.comosis learningradiology.com
Hernias
• Umbilical- in kiddos, close spontaneously by age 2. In
adults: 2/2 obesity, ascites or pregnancy.
• Indirect Inguinal- MC  through inguinal ring (lat to
epigastric vessles) in spermatic cord. R>L, more often
congenital (patent proc vaginals)
• Direct Inguinal-  through Hasselbeck’s triangle
(med to epigastric vessles), more often acquired
weakness.
• Femoral- more common in women.
• Tx- emergent surgical repair if incarcerated to
avoid strangulation. Elective if reducible.
Inflammatory Bowel Disease
• Involves terminal ileum? Crohn’s. Mimics appendicitis. Fe deficiency.
• Continuous involving rectum? UC. Rarely ileal backwash but never higher
• Incr risk for Primary UC. PSC leads to higher risk of cholangioCA
Sclerosing Cholangitis?
• Fistulae likely? Crohn’s. Give metronidazole.
• Granulomas on biopsy? Crohn’s.
• Transmural inflammation? Crohn’s.
• Cured by colectomy? UC.
• Smokers have lower risk? UC. Smokers have higher risk for Crohn’s.
• Highest risk of colon cancer? UC. Another reason for colectomy.
• Associated w/ p-ANCA? UC.
Treatment = ASA, sulfasalzine to maintain remission. Corticosteroids to induce
remission. For CD, give metranidazole for ANY ulcer or abscess. Azathioprine,
6MP and methotrexate for severe dz.
IBD Images & Complications

commons.wikimedia.org
medinfo.ufl.edu/~bms5191/gi/images/cd1a.jpg

http://www.ajronline.org/cgi/con
tent-nw/full/188/6/1604/FIG20

studenthealth.co.uk
Diverticular Disease
• Diverticulosis-
– False diverticulae (only outpocketings of mucosa)
– Occur 2/2 low fiber diet in areas of weakness where blood
vessels penetrate  bleed
– Complications are bleeding, obstruction, diverticulitis
• Diverticulitis-
– Diverticulum becomes obstructed and forms
abscess/perforates
– LLQ pain, either constipation or diarrhea,
– Look for free air, CT is best imaging to
evaluate for abscess. No Barium enema!
– Tx w/ NPO, NG suction, IVF, broad spec abx & pain control. www.meddean.luc.edu/.../GI/Diverticulit
is2.jpg

– Do colonoscopy: 4-6 weeks later.


– Surgery indicated if: multiple episodes, age <50. Elective is better than
emergency (can do primary anastamosis)
Colorectal Cancer
• RF
– Genetics? AFP, Lynch Syndrome, HNPCC, Gardners, Cowdens
– Other? UC. Need colonoscopy 8-10yrs after dx
• Sxs
– Right sided cancer = bleeding ourwebdoctor.com

– Left sided cancer = obstruction


– Rectal cancer = pain/fullness, bleeding/obstruction
• Work up DRE, transrectal ultrasound (depth of invasion),
Colonoscopy! CEA to measure recurrance, CT for staging.
• Tx
– For colon- remove affected segments & chemo if node +
– For rectum- upper/middle 1/3 get a LAR, lower 1/3 gets an APR
(remove sphincter, permanent colostomy)
AAA
• Screening = men 65-75 who have ever smoked. Do abdominal U/S.

• Sxs = pulsatile abdominal mass.


• Tx conservatively if:
if <5cm and asymptomatic, monitor growth every 3-12mo.
• Surgery indicated if: >5cm, growing <4mm/yr
• Rupture =
– severe sudden abdomen, flank or back, shock, tender
pulsatile mass.
– 50% die before reaching the hospital.
• Post-op complications = #1 cause of death  MI
– Bloody diarrhea-Ischemic colitis
ASA
– Weakness, decreased pain w/ preserved vibr, prop- syndrome
– 1-2 yrs later if have brisk GI bleeding  Aortoenteric Fistula
Mesenteric Ischemia
• Acute Mesenteric Ischemia = surgical emerg!
– Acute abdominal pain in a pt w/ A-fib subtherapeutic
on warfarin or pt s/p high dose vasoconstrictors
(shock, bypass).
– Work up is angiography (aorta and SMA/IMA)
– Tx is embolectomy. If thrombus, or aortomesenteric
bypass.
• Chronic Mesenteric Ischemia =
– Slow progressing stenosis (req stenosis of 2.5 vessels
 Celiac, SMA and IMA).
– Severe MEG pain after eating, food fear and weight
loss. “Pain out of proportion to exam”.
– Dx w/ duplex or angiography.
– Tx w/ aortomeseteric bypass or transaortic mesenteric
endarterectomy.
Peripheral Artery Disease
• Acute arterial occlusion: 5P’s  no dopplerable pulses.
– Tx w/ immediate heparin + prepare for surgery.
– Surgery (embolectomy or bypas) done w/in 6hrs to avoid
loss.
– Thrombolytics may be possible if: no surg in <2wks,
hemorrhagic stroke.
– Complications = compartment syndrome during reperfusion
period  do fasciotomy watch for myoglobinuria.
• Claudication-
– Pain in butt, calf thigh upon exertion.
– Best test? Ankle-Brachial Index
– Normal- >1
– Claudication & Ulcers- 0.4-0.8, use medical management
– Limb ischemia- 0.2-0.4, surgery is indicated
– Gangrene <0.2, may require amputation
DVT and PE
• High risk after surgery (esp orthopedic)
• DVT-
– Dx w/ Duplex U/S & also check for PE
– Tx w/ heparin, then overlap w/ warfarin for 5 days, then download.imaging.consult.com/...
continue warfarin for 3-6mo. /gr1-midi.jpg

– Complications- post-phlebotic syndrome = chronic valvular


incompetence, cyanosis and edema
• PE-
– Random signs = right heart strain on EKG, sinus tach, decr
vascular markings on CXR, wedge infarct, ABG w/ low CO2 and
O2.
– If suspected, give heparin 1st! Then work up w/ V/Q scan, then
spiral CT. Pulmonary angiography is gold standard.
– Tx w/ heparin warfarin overlap. Use thrombolytics if severe but
NOT if s/p surgery or hemorrhagic stroke. Surgical
thrombectomy if life threatening. IVC filter if contraindications
to chronic coagulation.
Work up of a Thyroid Nodule
• 1st step? Check TSH
• If low? Do RAIU to find the “hot nodule”. Excise or radioactive I131
• If normal? FNA
• If benign? Leave it alone.
• If malignant? Surgically excise and check pathology
• If indeterminate? Re-biopsy or check RAIU
• If cold? Surgically excise and check pathology
– Papillary MC type, spreads via lymph, psammoma bodies
– Follicular Spreads via blood, must surgically excise whole thyroid!
– Medullary Assoc w/ MENII (look for pheo, hyperCa). Amyloid/calci
– Anaplastic 80% mortality in 1st year.
– Thyroid Lymphoma Hashimoto’s predisposes to it.
Work up of an Adrenal Nodule
• #1- check functional status
Diagnosis Features Biochemical Tests
Pheochromocytoma High blood pressure, Urine- and plasma-free
catechol symptoms metanephrines
Primary aldosteronism High blood pressure, low Plasma aldosterone-to-
K+, low PRA* renin ratio
Adrenocortical carcinoma Virilization or feminization Urine 17-ketosteroids
Cushing or "silent" Cushing Cushing symptoms or Overnight 1-mg
syndrome normal examination results dexamethasone test

• #2- if <5cm and non-function  observe w/


CT scans q6mo.
If >6cm or functional  surgical
excision
http://emedicine.medscape.com/article/116587-treatment
Parathyroid Disease
• Hypoparathryoidism
– Typically comes from thyroidectomy
– Sxs are perioral numbness, Chvortek, Trousseau
– ↓*Ca+, ↑*PO4+, ↓*PTH+
• Hyperparathyroidism-
– Usually asymptomatic ↑Ca, but can present w/ kidney stones,
abdominal or psychiatric sxs
– ↑*Ca+, ↓*PO4+, ↑vitD, ↑*PTH+
– Dx w/ FNA of suspicious nodules. Can use Sestamibi scan.
– Tx w/ surgical removal of adenoma. If hyperplasia, remove all 4
glands and implant 1 in forearm.
• MEN-
– MEN1- pituitary adenoma, parathyroid hyperplasia, pancreatic
islet cell tumor.
– MEN2a- parathryoid hyperplasia, medullary thyroid cancer,
pheochromocytoma
– MEN2b- medullary thyroid cancer, pheochromocytoma,
Marfanoid
Work up of a Breast Mass
• U/S can tell if solid or cystic. MRI is good for eval dense
breast tissue, evaluating nodes and determining
recurrent cancer.
– Best imaging for the young breast
– U/S good for determining fibroadenoma/cysto-sarcoma
phyllodes.
• Aspiration of fluid if cystic, FNA for cells if solid
– Send fluid for cytology if its bloody or recurs x2
– Fibrocystic change  cysts are painful and change w/
menses. Fluid is typically green or straw colored.
• Restrict caffiene, take vitamin E, wear a supportive bra
• Excisional biopsy if palpable or if fluid recurs
• Mammaographically guided multiple core biopsies
Breast Cancer
• RF: BRCA1 or 2, person hx of breast cancer, nulliparity, www.pathconsultddx.com/.../gr1-sml.jpg

endo/exogenous estrogen.
• DCIS-
– Either excision w/ clear margins or simple mastectomy if multiple
lesions (no node sampling) + adjuvant RT.
• LCIS-
– More often bilateral. Consider bilateral mastectomy only if +FH,
hormone sensitive, or prior hx of breast cancer
• Infiltrating ductal/lobular carcinoma-
– If small and away from nipple, can do lumpectomy w/ ax node
sampling. Adjuvant RT. Chemo if node +. Tamoxifen or Raloxifen if ER +
– Modified radical mastectomy w/ ax node sampling w/o adjuvant RT
gives same prognosis.
• Paget’s Dz-
– Looks like eczema of the nipple. Do mammogram to find the mass.
• Inflammatory-
– Red, hot, swollen breast. Orange peal skin. Nipple retratction.

riversideonline.com
Skin Cancer
• Basal Cell Carcinoma-
– Shave or punch bx then surgical removal (Mohs)
• Squamous Cell Carcinoma-
– AK is precursor lesion (tx w/ 5FU or excision) or http://emedicine.medscape.com/article/

keratoacanthoma. 276624-media

– Excisional bx at edge of lesion, then wide local excision.


– Can use rads for tough locations.
• Melanoma-
– Superficial spreading (best prog, most common)
– Nodular (poor prog) http://emedicine.medscape.com/article/1
101535-media

– Acrolintiginous (palms, soles, mucous membranes in darker


complected races).
– Lentigo Maligna (head and neck, good prog)
– Need full thickness biopsy b/c depth is #1 prog
– Tx w/ excision-1cm margin if <1mm thick,
2cm margin if 1-4mm thick, 3cm margin if >4mm
myhealth.ucsd.edu
– High dose IFN or IL2 may help
Sarcoma
• Soft Tissue Sarcoma-
– Painless enlarging mass. (Don’t confuse w/ bruised muscle.
– Dx w/ biopsy (NOT FNA). Excisional if <3cm otherwise
incisional.
– Tx w/ wide, local excision or ampulation + RT.
– Spreads 1st to the lungs (hematogenously)  can do
wedge resection if only met and primary is under control.
• Liposarcoma-
– 99% DON’T come from lipoma
• Fibrosarcoma/Rhabdomyosarcoma/
Lymphangiosarcoma-
– Hard round mass on extremity. Can occur in areas of
chronic lymphedema
Work up of a Neck Mass
• 7 days = inflammatory, 7 mo = cancer, 7 yrs =
congenital
– MC is a reactive node, so #1 step is to examine teeth,
tonsils, etc for inflammatory lesion
– If you find a lesion that’s still there in 2 week  FNA it!
– If node is firm, rubbery and “B sxs” are present 
excisional bx looking for Lymphoma
• Hodgkins = lymphocyte predom is good prog factor. Reed
Sternberg cells. lmp.ualberta.ca

• Non-Hodgkins = nodular and well-dif are good prog factor.


• Staging CT, CXR and laparotomy for chemo and XRT treatment
• If midline  thyroglossal duct cyst, move tongue 
mass moves. Remove surgically. cssd.us

• If anterior to SCM  brancial cleft cyst


• If spongy, diffuse and lateral to SCM  cystic hygroma
(Turners, Down’s, Klinefelters)
ENT Cancers
• Oral Cancer-
– Most freq squamous cell. In smokers & drinkers
– Tx w/ XRT or radical dissection (jaw/neck)
• Laryngeal Cancer-
– Laryngeal papilloma in kiddo w/ stridor or cough
– Squamous cell in adults.
– Tx w/ laryngoscope laser or resection
• Pleomorphic Adenoma-
atlasgeneticsoncology.org

– MC salivary glad tumor. Usually on parotid. Benign but recurs


• Warthlin’s Tumor-
– Papillary cystadenoma lymphomatosum. Benign on parotid
gland.
– Can injure facial nerve (look for palsy sxs in ? Stem)
• Mucoepidermoid Carcinoma-
– MC malignant tumor. Arises from duct. Causes pain and CNVII
palsy
Pedi-Surg
Baby is born w/ respiratory distress,
scaphoid abdomen & this CXR.
Diaphragmatic hernia
• Biggest concern? Pulmonary hypoplasia
emedicine.medscape.com
• Best treatment? If dx prenatally, plan
delivery at @ place w/
ECMO. Let lungs mature
3-4 days then do surg
Baby is born w/ respiratory
TE- Fistula
distress w/ excess drooling.
• Best diagnostic test? Place feeding tube, take xray, see it
coiled in thorax
GI disorders Gastroschisis
• Defect lateral (usually R) of *will see high
the midline, no sac. maternal AFP
– Assoc w/ other disorders? Not usually. bms.brown.edu

– Complications? May be atretic or necrotic req


removal. Short gut syndrome
Omphalocele
• Defect in the midline.
Covered by sac.
– Assoc w/ other disorders? Yes bms.brown.edu

Umbilical Hernia
• Defect in the midline. No
bowel present.
– Assoc w/ other disorders? Assoc w/ congenital hypo-
– Treatment? thyroidism. (also big tongue)
images.suite101.com/617141_c
om_picture067.jpg
Repair not needed unless persists past age 2 or 3.
A vomiting baby
• 4wk old infant w/ non-
Pyloric Stenosis
bileous vomiting and
palpable “olive”
– Metabolic complications? Hypochloremic, metabolic alkalosis
– Tx? Immediate surg referral for myotomy
• 2wk old infant w/ bileous
Intestinal Atresia
vomiting. The pregnancy Or Annular Pancreas
was complicated by poly-
hydramnios.
– Assoc w/? Down Syndrome (esp duodenal) Learningradiology.com

• 1 wk old baby w/ bileous


vomiting, draws up his legs, Malrotation and volvulus
*Ladd’s bands can kink the duodenum
has abd distension.
– Pathophys? Doesn’t rotate 270 ccw around SMA
Pooping Problems
• A 3 day old newborn has Meconium ileus- consider CF if +FH
still not passed meconium. *gastrograffin enema is dx & tx
Hirschsprung’s- DRE  explosion of poo.
– DDX? (name 2)
bx showing no ganglia is gold standard
• A 5 day old former 33
weeker develops bloody Necrotizing Enterocolitis
diarrhea
– What do you see on xray? Pneumocystis intestinalis (air in the wall)
– Treatment? NPO, TPN (if nec), antibiotics and resection of necrotic bowel
– Risk factors? Premature gut, introduction of feeds, formula.
• A 2mo old baby has colicky
abd pain and current jelly Intussusception
*Barium enema is dx and tx
stool w/ a sausage shapend
mass in the RUQ.
Urology
• BPH-
– Anticholinergics meds make it worse  foley for acute urinary
retention.
– Medical Tx 1st w/ tamsulosin or finasteride
– Surgical Tx w/ TURP (hyponatremia, retro-ejac)
• Prostate Cancer-
– Nodules on DRE or elevated/rising PSA means  transrectal
ultrasound and bx. Bone scan looks for blastic lesions.
– Tx w/ surgery, radiation, leuprolide or flutamide.
• Kidney Stones-
– CT is best test. If stone <5mm, hydrate and let it pass. If >5mm, do
shock wave lithotripsy. Surgical removal if >2cm.
• Scrotal Mass-
– Transilluminate, U/S, excision! (don’t bx). Know hormone markers!
• Testicular Torsion-
– Acute pain and swelling w/ high riding testis.
– Do STAT doppler U/S  will show no flow (contrast w/ epididymitis)
– Can surgically salvage if <6hrs. Do orchiopexy to BOTH testes.
Ortho
• Avascular Necrosis- img.medscape.com/.../329097-333364-4215.jpg

– In kids  Leg-Calve-Perthe’s dz in 4-5 y/o w/ a painless


limp and SCFE in a 12-13 y/o w/ knee pain or sickle cell pts
– In adults  steroid use, s/p femur fracture.
• Osteosarcoma-
– Seen in distal femur, proximal tibia
@ metaphysis, around the knee
– Codman’s triangle and Sunray appearance
• Ewing Sarcoma-
img.medscape.com/.../329097-333364-4215.jpg

– Seen at diaphysis of long bones,


night pain, fever & elevated ESR
– Lytic bone lesions, “onion skinning”.
– Neuroendocrine (small blue) tumor
www.learningradiology.com/.../cow279lg.jpg
Transplant
• Hyperacute Rejection-
– Vascular thrombosis w/in minutes
– Caused by preformed antibodies
• Acute Rejection-
– Organ dysfunction (incr GGT or Cr depending on organ)
w/in 5days – 3mo. Due to T-lymphocytes.
– Technical problems common in Liver  1st check for biliary
obstruction w/ U/S then check for thrombosis by Doppler.
– In heart, sxs come late, so check ventricular bx periodically.
– Tx w/ steroid bolus and antilymphocyte agent (OKT3)
• Chronic Rejection-
– Occurs after years. Due to T-lymphocytes.
– Can’t treat it. Need re-transplantation.
Anesthesia
• Local- (lidocaine, etc) To prevent systemic absorption  numb
– Why give with epi? tongue, seizures hypotension, bradycardia,
arrhythmias
– No epi where? Fingers, nose, penis, toes
• Spinal-Subarachnoid- (bupivacaine, etc)
– For ppl who can’t be intubated. Can’t give if incr ICP or hypotensive.
• Epidural- (local + opiod)
– If “high block”  blocks heart’s SNS nerves and phrenic nerve.
• General-
– Merperidine: Norperidine metabolite can lower seizure
threshold esp in pts w/ renal failure.
– Succinylcholine: Can cause malignant hyperthermia, hyperK (not
for burn or crush victim)
– Rocuronium, etc: Sometimes allergic rxn in asthmatics

– Halothane, etc: Can cause malignant hyperthermia (dantroline


Na), liver toxicity.
SURGERY SHELF REVIEW
Dr. Dominik Lipinski MD Surgery – Preoperative Care

––––––––––––––––– CV ISSUES ––––––––––––––––– Previous coronary angioplasty: high risk (33%) of Delirium in liver failure: possible causes include
coronary restenosis, so stress test is indicated; if CNS abnormality, electrolyte imbalance, GI
Goldman’s index: cardiac risk factors in surgery – angioplasty is recent, delay surgery for several bleeding, sepsis, or bacterial peritonitis; evaluate
JVD (#1), recent MI (#2), PVCs and arrhythmias, age weeks for mental status change, and tap the ascites
>70, emergency surgery, aortic stenosis
Angina: indicates coronary artery disease, evaluate Ascitic leakage: high risk of bacterial peritonitis;
JVD: indicates CHF, #1 CV risk factor overall, give for possible coronary revascularization manage by sending fluid to lab, giving IV
β-blockers, Ca2+-channel blockers, digitalis, and antibiotics, and urgent hernia repair
diuretics if possible before surgery AFib: give anticoagulation and β-blockers, and
cardiovert them to normal sinus rhythm Hemorrhoids: suspect portal HTN in pt with
MI: previous MI increases risk of post-op MI, so cirrhosis + hemorrhoids, high risk of hemorrhage
consult cards and order a stress test Carotid bruit: indicates carotid stenosis; indicated
for CEA if high grade (>70%) stenosis Malnutrition: indicated by recent weight loss (#1) or
MI within 30 days: very high risk, delay surgery albumin <3.0 (#2); give 7-10 days of pre-op nutritional
Previous stroke: order a carotid duplex study, in support ideally
Family Hx of MI: get a concentrated cardiac FHx, order to assess the carotid arteries
EKG, and exercise stress test to assess risk factors ––––––––––––––––– GU ISSUES –––––––––––––––––
Gangrenous toe: peripheral revascularization is
↑cholesterol: increased risk of coronary artery more urgent than a full cardiac work-up Dysuria: get urinalysis and urine culture; if positive
disease, but do not postpone surgery for UTI, delay surgery until resolved
––––––––––––– RESPIRATORY ISSUES ––––––––––––
Premature ventricular contractions: at risk of Chronic renal failure: delay surgery until pt is stable,
arrhythmia due to ventricular dysfunction, Smoking: up to 6× risk for post-op complications due dialysis started, and any other problems resolved
indicated for stress test and echo to compromised ventilation, must quit smoking for 2
months before surgery CRF × K+ measurement: needs to be obtained
Ejection fraction: SV/EDV; normal EF >67%, an immediately before surgery, since CRF can result
EF<35% increases risk of operative MI COPD: give bronchodilators and try to improve in rapid electrolyte imbalances
pulmonary status as much as possible
Diabetes: should be NPO 8 hours before surgery, CRF × operative bleeding: renal failure causes
administer IVF with D5, check glucose morning of Severe COPD: very high risk for acute pulmonary platelet dysfunction secondary to uremia;
surgery; if glucose >250 → 2/3 of insulin, if glucose failure with surgery; teach patient about give desmopressin or FFP, but not platelets
<250 → 1/2 of insulin incentive spirometry, give bronchodilators, and
mobilize post-op to prevent atelectasis CRF × operative hypotension: many possible
Hyperglycemia: ideal glucose is 100-250, causes, consider glucocorticoid (aldosterone)
delay surgery until glucose is under control Green sputum: give oral antibiotics, and schedule deficiency in a pt who has taken steroids before
surgery after Tx is complete
Diabetic coma: absolute contraindication to Kidney transplant pt: require perioperative steroids
surgery; give IVF, correct acidosis and glucose Bloody sputum: indicates active infection or lung
cancer; requires a full work-up including CXR, CT Post-op hyperkalemia: check EKG for peaked T
↓hct: important to determine underlying cause of scan, and bronchoscopy waves, Tx C BIG K DIe – calcium gluconate, bicarb-
anemia, consider possible colorectal cancer insulin-glucose, kayexalate, and dialysis
–––––––––––– HEPATOBILIARY ISSUES –––––––––––
↑hct: either hypovolemia or polycythemia; if –––––––––––––– VALVULAR ISSUES –––––––––––––
hypovolemic → delay until hydrated, if polycythemic Acute cholecystitis: presents as fever, RUQ pain,
→ important to determine underlying cause WBC>15; get U/S → IVF, abx, lap chole w/in 72 hours Mitral stenosis: elevated LA pressure can lead to
cor pulmonale (PH+RVH); management includes
Obesity: higher risk of HTN, cardiovascular disease, Child’s classification: stratifies risk of surgery in pts cards consult, prophyactic abx for endocarditis, can
post-op atelectasis, type 2 diabetes, DVTs; require with liver failure; measures 3 labs (albumin, bilirubin, go to surgery if stable
DVT prophylaxis and aggressive post-op pulmonary PT) and 3 clinical findings (encephalopathy, ascites,
care for preventing atelectasis nutrition) Mitral stenosis × CHF: high risk of mortality;
requires extensive cardiac work-up, EKG, echo,
HTN: diastolic BP >110 is high risk of CV complications, Child’s group A: 0-5% mortality and operative monitoring of cardiac status
β-blockers reduce overall risk
Child’s group B: 10-15% mortality Aortic stenosis: Sx triad of angina, dyspnea, syncope,
Atherosclerosis: can present as acute coronary and high possibility of sudden death; requires
syndome or peripheral vascular disease, always Child’s group C: >25% mortality; not good surgical cardiac work-up and operative monitoring
evaluate pt’s cardiac risk factors candidates until Child’s status is improved
Endocarditis prophylaxis: recommended for GI
CV evaluation: EKG (and compare to old EKG), Alcohol use: delay surgery until pt has undergone procedures, GU procedures, and HEENT procedures
persantine thallium stress test, dobutamine echo withdrawal, since post-op withdrawal syndrome has
a high risk of morbidity/mortality Cardiomyopathy: high risk of arrhythmias, CHF,
LBBB: indicates underlying ischemic heart disease heart failure, and sudden death; manage with cards
Liver failure: make sure the pt is in a compensated consult and full cardiac work-up
RBBB: indicates significant pulmonary disease, but state, abstain from alcohol for 6-12 weeks, control
can be normal in up to 10% of pts ascites, normalize nutrition status and coagulation –––––––––––––––––– GI ISSUES –––––––––––––––––
factors
Previous CABG: decreases cardiac risk if performed 6 Bowel prep: decreases fecal mass and bacterial
months to 5 years before surgery, effect on cardiac Pressure necrosis on hernia: high risk of rupture content in the colon → prevents colon surgery
risk unclear if >5 years with a high mortality rate, requires urgent repair
Surgery – Preoperative Care

complications; put pt on clears day before surgery,


NPO at midnight, and give a cathartic agent

GoLYTELY: an isotonic formula that causes no


change in electrolyte or water balance; fluid
remains in colon and causes a “volume washout”

Fleet’s Phospho-Soda: a sugar-rich, hypertonic


formula that works by drawing fluid into the GI
tract; risks include dehydration, metabolic
acidosis (loss of bicarb); avoid in diabetics

Magnesium citrate: an osmotic agent that draws


fluid into GI tract (mag is poorly absorbed); risks
include dehydration and hypermagnesemia; avoid
in renal failure

–––––––––––––––– OTHER ISSUES –––––––––––––––

Anesthesia of choice: multiple factors involved, so


it’s best to consult an anesthesiologist

Local anesthesia: ↓systemic effects, ↑pain

Spinal anesthesia: ↓pulmonary complications,


↓control over cardiac or vascular mechanisms

General anesthesia: ↑physiologic control,


↑pulmonary complications, ↓HR/BP

Aspirin use: stop aspirin use 7-10 days prior to


surgery, will affect platelet aggregation

NSAIDs use: stop NSAIDs 2 days prior, will affect


platelet aggregation

Cellulitis: active infections are associated with


higher risk of post-op wound infections; delay
surgery until infection is resolved
Surgery – Postoperative Care

––––––––––––––– POST-OP FLUIDS –––––––––––––– toilet and incentive spirometry (can develop into volume depletion (slow) or altered MS (rapid); give
pneumonia on POD #3 if left untreated) D5½NS to correct imbalance
Post-op fluid management: replace blood lost
during surgery + provide maintenance IVF + make up UTI: Dx urinalysis and urine Cx, Tx abx Hyponatremia: due to SIADH or isotonic fluid loss w/
for fluid loss in drains/NG tubes/fistulas free water resorption, presents as coma and
Urosepsis: UTI + septic shock; presents as cloudy convusions; Tx water restriction and LR/NS
Surgical blood loss: replace in a 3:1 ratio with IVF urine, fever, hypotension, and ∆mental status;
(e.g. replace 500mL blood loss with 1.5L NS or LR) Dx urinalysis and urine Cx, Tx empiric abx + IVF Hypokalemia: due to diarrhea or vomiting, give K+ at
a rate of <10 mEq/hr
Maintenance IVF: administer D5½NS+KCl, DVT: Dx doppler studies, Tx heparin
using the 100/50/20 rule for daily fluid Hyperkalemia: due to renal failure, aldosterone
requirements, or 4/2/1 rule for hourly fluids Wound infection: Dx examine wound site for blockers, or release from dead tissue (crush injury,
erythema and fluctuance; if cellulitis → abx only, ischemic bowel, etc.); Tx C BIG K DIe – calcium
Drains/NG tubes/fistulas: fluid loss should be if abscess → drain pus and BID dressing changes gluconate, bicarb-insulin-glucose, kayexalate, and
replaced milliliter for milliliter with regard to dialysis
specific electrolyte content IV infection: Tx remove catheter and
inflammation should resolve, all IV sites should be Metabolic acidosis: pH<7.4 + HCO3<24, Tx correct
Third space: fluid sequestered into ISF due to rotated every 4 days for prevention underlying problem (e.g. fluid resuscitation)
inflammation or injury, mobilized 3-5 days after
recovery → requires decrease in IVF rate Suppurative phlebitis: infected thrombus at site Metabolic alkalosis: pH>7.4 + HCO3>24, Tx KCl
of venipuncture; Tx remove catheter and surgical
––––––––– OPERATIVE COMPLICATIONS –––––––– excision of infected vein to the first non-infected Respiratory acidosis: pH<7.4 + CO2>40 due to
branch, leave wound open, and give IV abx hypoventilation, Tx mechanical ventilation
Malignant hyperthermia: T>104 following
anesthesia w/ high risk of myoglobinuria; Tx GI fistula: causes leakage of GI contents from wound Respiratory alkalosis: pH>7.4 + CO2<40 due to
dantrolene, 100% O2, and cooling blankets site; Tx NPO, TPN, and protect abdominal wall until hyperventilation caused by pain, fever, sepsis or
body can heal itself early ARDS; Tx correct underlying problem
Bacteremia: T>104 and chills within 1 hour of an
invasive procedure; get blood Cx ×3 and start Non-healing fistula: factors that prevent healing
empiric abx are FRIEND – foreign body, radiation, infection,
epithelialization, neoplasm, distal obstruction
Aspiration: complication of awake intubations;
leads to sudden death, chemical pneumonitis, or Fistula × peritonitis: requires surgical exploration
secondary pneumonia
Fistula × abscess: requires percutaneous drainage
Aspiration management: prevent via NPO and
antacids before intubating, Tx BAL and broncho- Post-op chest pain: consider MI or massive PE
dilators (steroids are useless)
MI: presents as chest pain +/- other classic signs;
Tension PTX: complication of intubation in weak or Dx CK-MB or troponin I, treat the complications
traumatized lungs, presents as “difficulty to bag”,
progressive hypotension and JVD; Tx emergent Massive PE: presents as chest pain, hypoxia, and
needle decompression + chest tube prominent JVD; Dx V-Q scan, Tx heparin → IVC
filter if PE recur while anticoagulated
––––––––––– POST-OP COMPLICATIONS ––––––––––
ARDS: presents as hypoxia due to septic shock, Tx
Normal urine output: at least 0.5-1 mL/kg/hr intubation w/ high PEEP and permissive hypercarbia,
then look for source of sepsis
Post-op urine retention: presents as a need to
void, but inability to do so; Tx straight cath at 6 Delirium tremens: presents as hallucinations in an
shours post-op and Foley after 2nd straight cath alcoholic POD #2-3, prevention is key and Tx is
controversial (benzodiazepines if choice is given)
UOP = 0: most likely a kinked or plugged Foley
Hepatic encephalopathy: presents as coma in a liver
UOP < 0.5: either fluid deficit (bleeding out) or failure pt s/p TIPS due to NH4+ toxicity
acute renal failure; give a bolus of 500mL IVF and
if UOP responds, then it’s due to fluid deficit Wound dehiscence: presents as salmon-colored fluid
soaking dressings s/p open laparatomy POD #5;
Post-op hematuria: consider bladder overdistention, stabilize wound site, surgical closure at a later date
cancer, infection, kidney stones, trauma, prostatitis,
and cyclophosphamide; get urology consult Evisceration: dehiscence + intestines spilling out;
keep pt in bed and cover bowel w/ sterile dressings,
Post-op fever: consider the five Ws – wind emergency surgical closure necessary
(atelectasis POD #1), water (UTI POD #3), walking
(DVT POD #5), wound infection (POD #7+), wonder ––––––––––– FLUIDS & ELECTROLYTES ––––––––––
drug (drug-induced fever)
Hypernatremia: every 3 Na+ over 140 indicates 1L
Atelectasis: partial lung collapse, Dx bilateral water loss (e.g. Na+ 146 = 2L water loss), presents as
inspiratory crackles, prevent with pulmonary
Surgery – Wound Healing

––––––––––––––– WOUND HEALING ––––––––––––– Prophylactic abx: recommended for any clean-
contaminated or contaminated procedures,
Primary intention: close approximation of wound insertion of prosthetic material, immunosuppression,
edges via sutures or staples or poor blood supply; give single dose 1 hour pre-op
and single dose post-op
Primary intention timing: epithelialized by POD
#2, max collagen in 5-7 days, avoid weight-lifting
for 4-6 weeks, complete healing by 6 months
(only 60-80% of original strength)

Secondary intention: contamined wounds left open


to prevent abscess formation; granulation tissue
forms first, then contraction via myofibroblasts,
then finally a delayed epithelialization

Third intention: wound initially left open, then


delayed primary closure afterwards

Secondary/tertiary intention timing: much more


delayed than healing by primary intention

Healing growth factors: PDGF, TGF-β, FGF, EGF

Suture knot: hard, knot-like structure below wound


site; absorbable sutures will resolve with time, non-
absorbable sutures can be taken out under lido/epi

Stitch abscess: infected suture that intermittently


drains pus, should be taken out under lido/epi

Incisional hernia: presents as bulging at wound site


with increased abd pressure (e.g. coughing), needs
to be repaired surgically

Red/sensitive scar: assuming no wound infection, it


could be completely normal; observe for 6 months
before considering surgical revision

Hypertrophic scar: raised scar within site of incision;


observe until scar is stable, then steroids + excision

Keloid: raised scar that extends beyond site of


incision; will grow back if excised

–––––––––––––– WOUND INFECTION ––––––––––––

Wound infection: red and tender area on incision


site; Tx drainage and BID wet-to-dry dressing
changes, no antibiotics unless cellulitis is spreading

Post-infection management: observation is slow


and can result in contractions; other options
include skin grafting (must have <105 bacteria per
gram of tissue) or healing by third intention

Wound types: clean, clean-contaminated,


contaminated, infected

Clean wounds: no entry into GI, GU, or


respiratory tracts; <1% infection rate, Tx primary
closure

Clean-contamined wounds: enters GI, GU, or


respiratory tract in a controlled manner;
<10% infection rate, Tx primary closure

Contaminated wounds: major wound


contamination (e.g. bowel spillage or stab
wounds); Tx secondary closure
Surgery – Cardiothoracic Surgery

–––––––––––––––– LUNG CANCERS –––––––––––––– –––––––––––––––– LUNG DISEASES –––––––––––––– Off-bypass coronary surgery: grafting of ITA or GSV
to coronary arteries without bypass in order to avoid
Coin lesion: solitary pulmonary nodule on CXR; Pleural effusion: fluid build-up in pleural space, complications of bypass (e.g. general inflammatory
DDx primary lung cancer, granuloma (TB or fungal), cancer in older pt until proven otherwise; DDx CHF, response); reserved for high-risk pts
hamartoma, metastatic cancers bronchogenic carcinoma, mesothelioma, empyema,
pneumonia, TB effusions Dilated cardiomyopathy: dilation of myocardium
Coin lesion management: first thing is always to causes systolic dysfunction leading to progressive
get an older CXR for comparison studies → if Pleural effusion management: get thoracentesis dyspnea and fatigue, often preceded by viral URI
suspicious, sputum cytology and CT scan and culture pleural fluid, and a pleural biopsy
DCM Px: 1/3 recover, 1/3 stay the same, 1/3 die
Benign coin lesions: calcification = granuloma, Spontaneous PTX: rupture of apical blebs in healthy
bull’s-eye shape, popcorn shape = hamartoma, young people causes sudden chest pain and dyspnea; DCM Tx: β-blockers will improve heart function,
air-crescent sign = aspergilloma, Southwest Tx thoracostomy on water-seal drainage but heart transplant is indicated (requires
region = coccidioidomycosis, Ohio river valley = chronic immunosuppression)
histoplasmosis Recurrent/persistent PTX: indicated for thorascopic
excision of blebs, and pleurodesis (either scraping or Heart transplant: remove donor heart and isolate in
Indeterminate coin lesions: resection indicated talc application) causing pleural adhesions hypothermic cardioplegia solution, remove recipient
heart on bypass, suture donor heart in recipient;
Malignant coin lesions: spiculations or 20+ pack- Bilateral PTX: dangerous situation, indicated for MCC of death are infection due to immuno-
year smoking = primary lung cancer, multiple pleurodesis to prevent future recurrences suppression, and accelerated coronary artery
lesions = metastatic cancer; resection indicated atherosclerosis
Empyema: collection of pus within pleural cavity;
Lung cancer management: first thing is always CXR presents with cough, fever, chest pain, and pleural Constrictive pericarditis: presents as dyspnea,
→ if suspicious, sputum cytology and CT scan → if effusion on CT scan; MCC is Staph aureus hepatomegaly, and ascites due to diastolic failure;
still suspicious, bronchoscopy and mediastinoscopy Dx cardiac cath shows “square root sign”, Tx
w/ biopsy (“tissue is the issue”) Empyema Tx: initiate appropriate antibiotics, surgical correction
insert chest tube, evacuate pus collection, then
NSCLC staging: stage I is local, stage II involves hilar re-expand the lung ––––––––––––– VALVULAR DISEASES ––––––––––––
LN, stage III involves distal LN, stage IV is metastatic
cancer ––––––––––––––– HEART DISEASES –––––––––––––– Valvular disease management: first detected on
auscultation → get an echocardiogram
NSCLC Tx: stage I/II → first get spirometry and Unstable angina: progressive ischemia of
V/Q scans to see if pt is a surgical candidate, myocardium resulting in increasing angina at rest Valve replacement: mechanical valves (durable
then do pneumonectomy or sleeve lobectomy; and an ischemic EKG pattern but require anticoagulation) and porcine valves
stage III/IV → chemotherapy and radiation (nonthrombogenic but will deteriorate over time)
Unstable angina management: cardiac catheter
Surgery contraindications: FEV1 <800mL, and evaluate for potential revascularization, Tx Aortic stenosis: angina, dyspnea, syncope, and high
mediastinal LN involvement, distal metastases bypass or angioplasty risk of sudden death; Dx harsh mid-systolic murmur,
MCC is congenital bicuspid aortic valve with
SCLC staging: limited stage involves ipsilateral lung Ejection fraction: SV/EDV; normal EF is 67%, EF <50% dystrophic calcification, Tx valve replacement when
only, extensive stage involves contralateral lung is abnormal, EF <35% is high-risk for operative MI symptomatic

SCLC Tx: chemotherapy and radiation Coronary artery disease: blockage of coronary Aortic regurgitation: presents as loud, blowing
arteries, high risk of sudden death due to ventricular diastolic murmur and bounding pulses; if chronic →
Pancoast tumor: superior sulcus tumor at the apex arrhythmias; Tx bypass or angioplasty valve replacement when symptomatic, if acute →
of the lung; presents as Horner syndrome – ipsilateral emergent valve replacement and long-term abx
ptosis, miosis, anhidrosis, flushing due to loss of Left main disease: blockage of only LCA, worst Px
sympathetic tone (invades superior cervical ganglia) Mitral stenosis: presents as dyspnea and fatigue,
Three-vessel disease: blockage of LAD/RCA/LCX caused by rheumatic fever, results in cor pulmonale
Pancoast tumor Tx: two stages – irradiation for 6 (RVH+PH) and AFib; Tx valve repair or replacement
weeks to shrink tumor, then surgical resection Coronary artery bypass: open via median
sternotomy, isolate internal thoracic artery (best Mitral regurgitation: presents as dyspnea and
Bronchial adenoma: tumors that arise from within choice) or greater saphenous vein, put on bypass fatigue, caused by MVP, Dx high-pitched holosystolic
bronchi and cause obstruction; MC lung cancer in a and stop the heart, sew grafts to arteries, restart the murmur; Tx annuloplasty (preferred) or valve
nonsmoker under 30 y/o, Sx hemoptysis and heart and close up replacement
atelectasis, Dx CT scan and bronchoscopy (beware
of possibility of bleeding) CABG mortality: 3% overall, greater in high-risk Mitral prolapse: eccentric closure of mitral leaflets;
pts but they benefit the most from the surgery common in women, precedes severe disease in men
Bronchial adenoma Tx: lobectomy is curative
Coronary angioplasty: insertion of stent to prolong ––––––––––– MEDIASTINAL DISEASES –––––––––––
Mesothelioma: cancer arising from pleural patency of clogged coronary arteries, 33% rate of
mesothelium, shipyards and asbestos exposure are restenosis in 1 year Anterior mediastinal mass: thymomas, lymphomas,
risk factors, Dx CT scan shows thick pleural walls teratomas, other metastatic germ cell neoplasms
Cardioplegia solution: solution used to stop heart
Mesothelioma Px: not responsive to medical mid-diastole to protect it from ischemia and provide Thymoma: often presents with progressive
therapy, most pts have <1 year to live w/o surgery a motionless field; often used with hypothermia to muscle weakness secondary to myasthenia gravis,
prolong time of safe ischemia (up to 2.5 hours) Tx surgical removal via median sternotomy
Mesothelioma Tx: extrapleural pneumonectomy
is curative, but high rate of morbidity/mortality Lymphoma: Tx radiation and chemotherapy
Surgery – Cardiothoracic Surgery

Teratoma: contains hair and teeth, Tx surgical


removal via median sternotomy

Middle mediastinal mass: lymphomas, bronchogenic


cysts, pericardial cysts

Bronchogenic cysts: develop from foregut


remnants, Tx surgical removal via thoracotomy

Pericardial cysts: typical water bottle appearance,


Tx surgical removal via thoracotomy

Posterior mediastinal mass: neurogenic tumors


(MCC is neurilemoma)

Neurilemoma: dumbbell-shaped tumor found


adjacent to vertebral bodies, develops from
nerves and nerve sheathes, Tx surgical removal
via thoracotomy
Surgery – Vascular Surgery

–––––––––––––– CAROTID DISEASES ––––––––––––– Ankle-brachial index: ratio of BP in arm vs. leg; AAA: presents as painless pulsatile abdominal mass;
ABI >1.0 is normal, ischemic ABI is 0.6-0.8 management is getting ultrasound and CT scan, then
Atherosclerosis: systemic disease that has multiple (claudication) or 0.3-0.5 (rest pain) elective graft repair if aorta is >5 cm
manifestations (stroke, TIA, CAD, MI, mesenteric
ischemia, AAA, PVD); presence of any one of these Doppler tracing: normally triphasic due to systolic AAA repair post-op: major fluid shifts (third-space
should raise suspicion for the others flow → elastic recoil → diastolic flow; ischemic loss on POD#1/2, third-space mobilization on
signal is either biphasic (mild) or monophasic (severe) POD#3), cardiac problems due to aortic clamping,
Stroke: a neurologic deficit that doesn’t resolve and impotence due to damage of hypogastric
within 24 hours, Tx t-PA only within 3 hours Claudication: exercise-induced ischemia that causes circulation or autonomic nerves around IMA
reversible calf pain, MCC is an atherosclerotic SFA
Stroke management: carotid duplex study, Ruptured AAA: presents as pulsatile mass, back and
observation for improvement; CEA is not Claudication management: if mild → not abdominal pain, and hypotension; if unstable → OR,
indicated until pt has stabilized indicated for surgery, so exercise and lifestyle if stable → get CT scan or ultrasound, then OR
changes are recommended; if severe → get
Transient ischemic attack: brief neurologic deficit Doppler tracing, then arteriogram to localize Ruptured AAA repair: clamp aorta at level of
that fully resolves within 24 hours; due to thrombo- diaphragm to stop bleeding, then proceed with
embolus from internal carotid arteries Claudication Px: 1/3 improve on a non-op exercise repair; 80% risk of death by bleeding out in the OR
program, 1/3 stay the same, 1/3 get worse
TIA management: physical exam (check carotid AAA repair × bloody diarrhea: indicates ischemic
bruids, neuro, cardiovascular) + carotid duplex; Aortoiliac stenosis: presents as triad of claudication colitis due to interrupted IMA; Dx sigmoidoscopy,
Tx aspirin or carotid endarterectomy + absent femoral pulse + impotence Tx bowel rest if limited to mucosa, colectomy and
colostomy if full-thickness involvement
TIA × amaurosis fugax: emboli from carotid Aortoiliac stenosis Tx: depends on case;
travels to retina causing transient blindness; bilateral loss of femoral pulse → aortofemoral AAA repair × delayed fever: indicates vascular
Dx fundoscopic exam reveals Hollenhorst plaque, bypass graft, single segment iliac stenosis → graft infection via S. aureus or S. epidermidis;
a bright shiny spot in a retinal artery angioplasty, high risk pt → lifestyle changes Dx CT scan, Tx graft removal, debridement, and
IV antibiotics
TIA × aphasia: emboli from left carotid travels to Severe claudication: presents as claudication + rest
speech center located in left hemisphere pain or foot ulcers, common in diabetics; get a AAA repair × UGI bleed: indicates aortoduodenal
vascular work-up and arteriogram to determine level fistula, usually a small bleed followed 1-2 days
CEA: indicated for >70% carotid stenosis with either of occlusion, and assess general medical status later by massive bleeding; Dx CT scan, Tx graft
neurologic sx or asymptomatic bruits; complications removal and GI repair
include 1-3% risk of perioperative stroke, or injury to SFA stenosis Tx: reverse GSV to SFA graft
facial, vagus, or hypoglossal nerves Mesenteric ischemia: presents as postprandial abd
Iliac stenosis Tx: surgical revascularization or pain, weight loss, and multiple abd bruits due to
Post-CEA management: start aspirin, encourage balloon dilation atherosclerosis of celiac trunk or SMA
lifestyle modifications (e.g. diet and exercise, quit
smoking) since patient is still high-risk for MI SFA + iliac stenosis Tx: both of the above, either Mesenteric ischemia management: Dx
at the same time or sequentially (iliac first) mesenteric angiogram, Tx revascularization;
Asymptomatic carotid bruit: get a carotid duplex follow-up with aspirin and evaluation for other
study, indicated for CEA if >70% stenosis SFA + popliteal stenosis Tx: femoropopliteal atherosclerotic diseases
bypass to the best artery continuous with the
–––––––––––– PERIPHERAL DISEASES ––––––––––– foot, failure likely in distal and severe disease Aortic dissection: presents as acute onset tearing
chest/back pain due to severe HTN (200/140), CXR
Subclavian steal syndrome: presents as nothing at Multiple obstructions Tx: reconstruction may not shows widened mediastinum
rest, but arm claudication and CNS sx with arm be possible, limb amputation indicated
activity due to subclavian artery atherosclerosis; Dx Aortic dissection management: Dx MRI, spiral CT,
arteriogram, Tx bypass surgery Bypass pre-op: 10% risk of MI, arrhythmias, or heart transesophageal echo, or arteriography; if
failure; requires evaluation of cardiac status (get ascending aorta → go to OR for surgical repair, if
Arterial embolus: presents as the 6 P’s – pain, pallor, dipyridamole-thallium scintigraphy to rule out CAD) descending aorta → β-blockers
parasthesias, poikilothermia, pulselessness, paralysis;
clot source is usually Afib or recent MI Pre-op reveals <30% ejection fraction: high risk of –––––––––––––– VENOUS DISEASES –––––––––––––
post-op heart failure
Embolus Tx: requires urgent revascularization Deep venous thrombosis: presents as acute onset
within 6 hours; give heparin and go to OR for Pre-op reveals CAD: cancel or delay surgery, dull leg pain, unilateral swelling, and Homans’ sign
balloon catheter embolectomy +/- fasciotomy perform a less invasive procedure, pre-op CABG
or PTCA, or intensive intraoperative monitoring DVT risk factors: Virchow’s triad – stasis,
Embolus post-op: start warfarin for anti- endothelial damage, and hypercoagulability
coagulation, then get CT scan or aortography to Pre-op reveals recent MI: delay surgery for 3 mo
search for source of emboli DVT management: Dx duplex ultrasound, Tx
Bypass post-op: daily duplex studies to check for therapeutic heparin or LMWH, followed by long-
Compartment syndrome: revascularization of graft stenosis, give aspirin, educate on lipid control term warfarin therapy (follow INR)
acutely ischemic limb results in ischemia-reperfusion and foot care; MCC death is CAD
injury, causing muscle edema within fascial compart- Post-thrombotic syndrome: occurs in 10% of DVT pts,
ment and the 6 P’s Trash foot: post-op embolization of atherosclerotic presents as severe leg edema and ulceration around
debris following bypass results in cyanotic toe; ankle area due to chronic venous HTN
Compartment syndrome Tx: fasciotomy Tx heparinization of clot and long-term aspirin use

–––––––––––– ABDOMINAL DISEASES –––––––––––


Surgery – Vascular Surgery

Post-thrombotic syndrome management:


prevention via chronic use of support hose, heal
ulcers if they’ve already developed

DVT prophylaxis: indicated if pt is at risk for DVT;


includes leg compression devices and subcutaneous
low-dose heparin

DVT prophylaxis for hip fx: fondaparinux and leg


compression devices

Pulmonary embolus: presents as acute onset chest


pain, dyspnea, and hyperventilation (↓PCO2), due to
DVT embolization into lung → wedge-shaped area of
lung infarction, S1Q3T3 on EKG

PE management: get EKG to rule out MI, ABGs


(shows ↓PCO2), CXR, and pulse oximeter;
mismatch on a V/Q scan is diagnostic, Tx identical
to DVT Tx

Recurrent PE: due to failure of heparin therapy;


indicated for IVC interruption with a metal filter

Anticoagulation: therapeutic/high-dose heparin or


LMWH for Tx DVT/PE, low-dose for prophylaxis;
IVC filters when anticoagulation fails or is contra-
indicated

Anticoagulation × HIT: rare heparin side-effects


include thrombocytopenia and paradoxical
arterial clots, d/c heparin immediately

Anticoagulation × skin necroses: rare side-effect


of warfarin therapy, start warfarin along with
heparin to prevent this from happening

Anticoagulation × UGI bleed: life-threatening


condition that requires immediate d/c, give pt a
metal filter instead for PE prophylaxis

Phlegmasia cerulea dolens: presents as acute onset


leg edema with pain and cyanosis due to venous
outflow obstruction, high risk of nerve damage and
venous gangrene

PCD management: elevate leg and immediate


anticoagulation, then order duplex ultrasound
and pelvic CT scan to confirm the Dx

–––––––––– OTHER VASCULAR DISEASES –––––––––

Temporal arteritis: presents as severe, unilateral


headache, visual changes, and nodularity of
temporal artery; give high-dose steroids right away
to prevent blindness
Surgery – Upper GI Surgery

–––––––––––– ESOPHAGEAL CANCERS ––––––––––– Pain × PMHx: gallstones or alcoholism = acute PUD surgery indications: intractability,
pancreatitis (order amylase/lipase levels), NSAIDs perforation, obstruction, bleeding
Dysphagia: management includes barium swallow or steroid use = PUD
and esophagoscopy w/ biopsy; DDx achalasia, Duodenal ulcers: caused by ↑acid secretion; most
Zenker diverticulum, esophageal cancers, strictures Esophageal varices: present as UGIB due to portal commonly in 1st part of duodenum, DU in 2nd-4th part
HTN, often alongside coagulopathy (liver failure) indicates Zollinger-Ellison syndrome (gastrinoma)
Esophageal cancer: SCC in upper 2/3 due to cigs and
EtOH, adenocarcinoma in lower 1/3 due to Barrett’s, EV management: band the bleeding varices, DU types: posterior ulcers bleed due to gastro-
Sx progressive dysphagia and weight loss correct any coagulopathy, IV octreotide to lower duodenal artery, anterior ulcers perforate
portal pressure → if bleeding continues, repeat
EC management: esophagoscopy and biopsy, endoscopic banding → if bleeding continues, DU Tx: triple therapy → if ulcer persists, surgery
then staging via endoscopic ultrasound and CT TIPS or gastric balloon tamponade is indicated (HSV > TV+P >> TV+A), also get serum
scan, then specific Tx gastrin levels to rule out Z-E syndrome
EV follow-up: β-blockers to lessen chance of
EC staging: stage I invades submucosa only, stage rebleeding; good liver function → elective TIPS, Perforated DU: CXR shows free air under
II invades muscularis +/- LN, stage III invades bad liver function → liver transplant diaphragm; use omentum to patch the ulcer
adventitia + adjacent structures, stage IV is (Graham patch), then stop acid w/ PPIs or HSV
metastatic Mallory-Weiss syndrome: presents as UGIB due to
retching lacerating the lower esophagus, bleeding Perforated DU × sepsis: complete Graham patch
EC prevalence: adeno MC in US, SCC elsewhere usually stops spontaneously as soon as possible, give PPIs and IV abx, then
monitor in ICU and plan surgery for a later date
EC Tx: upper 1/3 – chemo and radiation only; Boerhaave syndrome: presents as epigastric pain
middle 1/3 – chemo and radiation to shrink the and fever due to retching perforating the esophagus; Bleeding DU: due to posterior ulceration into
tumor, then esophagectomy; lower 1/3 – Dx contrast swallow, Tx emergent surgical repair GDA; Tx oversew the ulcer, then stop acid w/ PPIs
esophagectomy and proximal gastrectomy; never
operate on stage IV cancer Instrumental perforation: presents as retrosternal Gastric ulcers: caused by ↓mucosal protection; type I
pain, fever, and pneumomediastinum s/p upper GI – lesser curvature at incisure, type II – duodenum
Advanced esophageal cancer: Sx severe dysphagia endoscopy; Dx contrast swallow, Tx surgical repair and stomach, type III – pylorus, type IV – GE junction
and chronic cough (due to aspiration from TE fistula),
management is palliative care b/c surgery won’t help GERD: presents as heartburn, regurgitation, GU Tx: PPIs → if ulcer persists after six weeks, do
waterbrash (sour taste), dysphagia, cough endoscopy and multiple marginal biopsies for
––––––––––– ESOPHAGEAL DISEASES ––––––––––– possibility of gastric cancer; if ulcer persists after
GERD Tx: PPIs → if it persists after six weeks, do 18 weeks, surgery is indicated
Zenker diverticulum: pulsion diverticulum that EGD w/ biopsy to see what’s going on
develops at upper esophagus due to abnormal GU surgery: wedge resection or distal
coordination of cricopharyngeal constriction; Normal EGD: PPIs or elective lap Nissen gastrectomy (due to possibility of cancer), TV+P
Sx dysphagia, regurgitation, and bad breath for types II and III due to ↑acid production
Esophagitis: multiple, nonulcerating erosions in
Zenker management: Dx barium swallow, Tx stomach; mild to moderate esophagitis → PPIs Bleeding GU: Tx excision rather than oversewing
cricopharyngeus myotomy for 8-12 weeks, severe esophagitis → lap Nissen,
uncontrollable bleeding esophagitis → subtotal Gastric cancers: lymphoma, adenocarcinoma, GIST
Traction diverticulum: diverticulum at middle gastrectomy (e.g. sarcomas and lipomas)
esophagus due to LN traction, indicates cancer
Barrett esophagus: intestinal metaplasia of Gastric adenocarcinoma: often spreads to left
Esophageal motility disorders: achalasia (only one esophageal epithelium; no dysplasia → Tx PPIs or supraclavicular (Virchow’s) node and ovaries;
treated surgically), nutcracker esophagus, spasms, lap Nissen; low-grade dysplasia → Tx lap Nissen + Tx proximal → total gastrectomy, if distal →
hypertensive LES annual surveillance; high-grade dysplasia → Tx distal gastrectomy w/ anastomosis; take out D1
esophagectomy LN at lesser curvature
Achalasia: hypertonic and non-relaxing LES w/
poorly relaxing esophagus, Sx dysphagia of Lap Nissen fundoplication: wraps fundus of stomach Linitis plastica: infiltrating carcinoma with desmo-
liquids > solids around LES to keep it in abdominal cavity (must plastic reaction causing stomach to look fixed
check for intact esophageal peristalsis first); contra- and rigid; Tx total gastrectomy w/ splenectomy
Achalasia management: Dx bird’s beak on barium indicated in morbidly obese (gastric bypass instead)
swallow and ↑LES pressure on manometry, Tx Gastric lymphoma: first determine cancer stage
Heller myotomy Hiatal hernia: hernia from abdominal cavity, through (CT scan, LN biopsy, and bone marrow Bx);
diaphragm, into chest cavity Tx partial thickness → radiation, full thickness →
Nutcracker esophagus: painful swallowing due to surgical resection
high amplitude action potentials, Tx nifedipine Type 1 hiatal hernia: sliding hernia, risk of reflux
esophagitis, Tx PPIs GIST: any soft tissue tumor of stomach; Tx wedge
Diffuse esophageal spasms: uncoordinated 3° resection w/ 1 cm negative margins (no LN)
peristalsis, Tx medically Type 2 hiatal hernia: paraesophageal hernia, risk
of incarceration/strangulation, Tx surgical repair Gastric varices: present as UGIB due to portal HTN,
Hypertensive LES: high LES pressure at baseline Tx uncontrollable bleeding w/ TIPS or splenectomy
but relaxes with swallow, Tx medically Type 3 hiatal hernia: presence of both types 1+2, instead of banding
Tx surgical repair
Acute epigastric pain: DDx acute pancreatitis, GERD,
PUD (gastric ulcers or duodenal ulcers), cholelithiasis, –––––––– GASTRIC/DUODENAL DISEASES ––––––––
gastroenteritis
Peptic ulcer disease: gastric ulcers, duodenal ulcers
Surgery – Pancreatic/Hepatic Surgery

––––––––––––– GB/BILIARY DISEASES –––––––––––– –––––––––––––––––– JAUNDICE ––––––––––––––––– Chronic pancreatitis: presents as constant epigastric
pain, steatorrhea, and diabetes in a chronic alcoholic;
Biliary disease progression: cholelithiasis → biliary Jaundice: elevated bilirubin and yellowing of skin; Tx insulin and pancreatic enzyme replacement
colic → acute cholecystitis; choledocholithiasis → three types – hemolytic, obstructive, hepatocellular
ascending cholangitis or gallstone pancreatitis ––––––––––––––– LIVER DISEASES ––––––––––––––
Hemolytic jaundice: ↑bilirubin (direct <20%),
Cholelithiasis: presents as nausea, vomiting, RUQ search for what’s killing the RBCs Liver cyst: simple cyst, leave it alone
pain w/o fever; only 15-20% are symptomatic
Hepatocellular jaundice: ↑bilirubin (direct 20-50%) Echinococcal cyst: multilocular cyst w/ calcified walls
Cholelithiasis in pregnancy: manage non-op if and ↑AST/ALT, consider HBV/HCV and alcoholism due to parasite Echinococcus granulosus; inject
possible (hydration and pain meds), elective lap hypertonic saline inside cyst and carefully excise it
chole can be done after delivery Obstructive jaundice: ↑bilirubin (direct >50%) and
↑alk phos, caused by CBD stones and cancers Liver abscess: multiple/small bacterial abscesses →
Biliary colic: fatty meal → CCK release → gall IV antibiotics; single/large bacterial abscess → perc
bladder contraction against non-lodged stone Painless jaundice: caused by biliary obstructive drain; amebic abscess (Mexicans) → metronidazole
resulting in transient RUQ pain for <6 hours; labs are tumors (ampullary cancer, duodenal cancer,
usually normal if episode has passed cholangiocarcinoma, pancreatic adenocarcinoma) Liver cancers: hepatic adenoma, focal nodular
hyperplasia, hemangioma, hepatoma
Cholecystitis: presents as fever, WBC>15, RUQ pain Painless jaundice management: Dx CT scan then
>6 hours; MC bacteria are E. coli, Bacteroides fragilis, ERCP, Tx Whipple if no mets or local invasion Hepatic adenoma: often presents as hypovolemic
Klebsiella, Enterococcus shock and distended abdomen, related to OCPs
Painless jaundice × occult bleed: indicates and anabolic steroid abuse; Tx d/c OCP → if it
Cholecystitis management: Dx abdominal U/S, ampullary cancer, get CT scan then Whipple persists, resect due to possibility of rupture
order CBC and LFTs, Tx lap chole within 72 hrs
–––––––––––– PANCREATIC DISEASES ––––––––––– FNH: Dx central stellate scar or sunburst pattern
Cholecystitis in elderly: elderly pts respond to on CT scan, no OCP relationship, leave alone even
sepsis with hypothermia and ↓WBC Pancreatic head mass: check for metastases, then if symptomatic
perform Whipple if no mets
Cholecystitis antibiotics: ciprofloxacin (Cipro) Hemangioma: leave alone even if symptomatic,
and metronidazole (Flagyl) to cover GNR and Pancreatic adenocarcinoma: presents as obstructive never spontaneously rupture, do not needle
anaerobes; abx not indicated for cholelithiasis jaundice, get a CT scan; cancer in head → Tx Whipple, biopsy due to possibility of rupture
in body or tail → distal pancreatectomy, mets or
Cholecystectomy indications: symptomatic local invasion → palliative care (no surgery) Hepatoma: presents as vague RUQ pain and mass
cholelithiasis, acute cholecystitis, and cholangitis; related to HBV/HCV and cirrhosis w/ ↑αFP; Dx CT
do not operate on asx stones Acute pancreatitis: presents as epigastric pain scan then Tx resection w/ negative margins
boring through to the back w/ ↑lipase/amylase; indicated as long as there’s no mets
Cholecystectomy complications: nicking the CBD amylase levels do not correlate with severity
(jaundice) or right hepatic artery (hepatitis) Portal HTN: Sx esophageal varices, caput medusa,
Pancreatitis causes: I GET SMASHED – Idiopathic, hemorrhoids; Tx TIPS (connect portal vein to hepatic
Post-op biliary leak: presents as Charcot’s triad; Gallstones (#1), EtOH (#2), Trauma, Steroids, vein to relieve pressure) as a “bridge to liver txp”
get an U/S and HIDA scan → Tx biliary drainage Mumps, Autoimmune, Scorpion sting, Hyper-
and temporary stent during ERCP triglyceridemia (#3), Hypercalcemia, ERCP, Drugs

Post-op obstruction: presents as Charcot’s triad Edematous pancreatitis: get amylase/lipase,


due to stricture or retained stone; get an U/S and then NPO, IVF, pain meds
HIDA scan → Tx biliary drainage and choledocho-
jejunostomy Hemorrhagic pancreatitis: presents as MSOF,
ARDS, and hemodynamic instability; send to ICU
Choledocholithiasis: presents as transient jaundice for resuscitation and serial CTs
and ↑alk phos; Dx U/S shows dilated bile ducts +/-
ERCP, Tx lap chole w/ CBD exploration Ischemic pancreatitis: Dx no blood flow to
pancreas on contrast CT, Tx IV abx and resection
Acute cholangitis: presents as Charcot’s triad
(jaundice, fever, RUQ pain) or Reynold’s pentad Gallstone pancreatitis: if amylase returns to
(shock, ∆MS) due to choledocholithiasis normal → lap chole + cholangiogram; if
complicated → ERCP to remove stone
Acute cholangitis management: IVF, antibiotics,
and U/S → ERCP to decompress biliary tree → Pancreatic abscess: presents as septic shock 2 weeks
finally lap chole w/ CBD exploration after acute pancreatitis; Dx dynamic CT scan, Tx perc
drain + antibiotics
GB polyps: <2 cm observe, >2 cm take it out due to
risk of adenocarcinoma Pseudocyst: abdominal pain and early satiety 5
weeks after acute pancreatitis, Tx cystogastrostomy
GB adenocarcinoma: presents as mass in GB fossa; only if it’s symptomatic and has been present for 6+
Dx CT scan, Tx open chole + hilar LN resection + liver weeks (must get Bx w/ frozen epithelial section and
resection w/ negative margins see no epithelial lining before the ostomy since
epithelium indicates cancer)
Porcelain GB: dystrophic calcification of GB has 50%
risk of adenocarcinoma, take it out
Surgery – Lower GI Surgery

––––––––––– SMALL BOWEL DISEASES ––––––––––– Mesenteric ischemia × peritonitis: presents as Crohn’s × rectal disease: rare, indicated for
rebound tenderness, ↑WBC, fever, or metabolic subtotal colectomy and ileostomy
Small bowel obstruction: presents as colicky abd acidosis due to necrotic bowel; ex lap indicated
pain, nausea/vomiting, and constipation; MCC are Ulcerative colitis: inflammatory disease involving
adhesions from previous surgery and hernias Mesenteric ischemia × AFib: indicates emboli rectum and continuous proximal extension, Bx
shooting from left atrium to celiac trunk or SMA shows mucosal involvement + crypt abscesses +
SBO management: get KUB → Dx air-fluid levels pseudopolyps, Dx lead pipe sign on CT scan
and dilated loops; Tx NPO, NG suction, IVF, and Mesenteric ischemia × ↑hct: polycythemia due to
initially observation severe dehydration requires IV fluid resuscitation UC management: IV steroids and 5-ASA
(sulfsalazine) for acute flare-ups, annual
SBO × electrolyte imbalance: “hypokalemic, Mesenteric ischemia × CHF: ischemia may be colonoscopy for possibility of cancer
hypochloremic metabolic alkalosis” due to emesis secondary to a low-flow, nonocclusive state;
indicated for mesenteric vasodilation and UC × severe dysplasia: total proctocolectomy,
SBO × bloody diarrhea: indicates obstructive improve cardiac output ileal pouch formation, and ileo-anal anastomosis;
tumor or ischemic bowel; Dx sigmoidoscopy → no further cancer surveillance needed
observe if mucosal, resection if full-thickness Mesenteric ischemia × aortic dissection:
dissected aorta can occlude mesenteric vessels; Pouchitis: presents as fever, bloody diarrhea, and
SBO × flatus: indicates partial SBO since gas can Dx angiography, Tx surgical repair pain on defecation s/p ileal pouch formation for UC;
get through, more likely to resolve w/o surgery Tx metronidazole
Mesenteric ischemia × ↓BP: either ischemic
SBO × diarrhea: indicates partial SBO due to fecal bowel causing septic shock, or hypotension Toxic megacolon: presents as fever, bloody diarrhea,
impaction and severe constipation causing low-flow, nonocclusive ischemia pain, and abdominal distention in a pt w/ UC

SBO × inguinal hernia: requires urgent hernia Left colon necrosis: bowel resection → anastomosis TM management: get KUB for confirmation, then
repair to relieve strangulation if stable, otherwise colostomy and Hartmann pouch NPO, IVF, NG suction, IV steroids and abx;
improves → no surgery, stays the same or gets
SBO × melanoma: melanoma is the MC tumor Long segment necrosis: bowel resection → small worse → surgery necessary
that metastasizes to intestine, surgery indicated bowel syndrome requiring chronic TPN or transplant
since these don’t resolve spontaneously TM × perforation: shows free air on upright CXR,
Short segment necrosis: bowel resection → indicated for total colectomy and ileostomy w/
SBO × other cancers: surgery indicated since anastomosis, “second look” operation if bowel Hartmann pouch
these don’t resolve spontaneously viability is indeterminate
TM × impending perforation: shows pneumatosis
SBO × peritonitis: presents as rebound tender- Small punctate necroses: indicates multiple small on CT scan, indicated for surgery (see above)
ness, ↑WBC, fever, or metabolic acidosis due to emboli or low-flow state; bowel resection →
necrotic bowel; indicated for ex lap anastomosis, “second look” operation if bowel ––––––––––––––– APPENDICITIS –––––––––––––––
viability is indeterminate
SBO × adhesions: indicated for ex lap → lysis of Appendicitis: presents as RLQ pain, low grade fever
adhesions Bowel ischemia but no necrosis: try to revascularize and leukocytosis; MCC lymphoid hyperplasia
the bowel via removing or bypassing the occlusion
SBO × closed loop obstruction: usually due to Appendicitis management: get CT scan or U/S
adhesive band occluding two segments of bowel; Low-flow but no necrosis: non-op management of (U/S is cheaper); if uncomplicated → app’y, if
indicated for ex lap → lysis of adhesions, hemodynamic status, surgery should be avoided abscess (↑↑fever/↑↑WBC) → perc drain, interval
resection of any dead bowel, and “second look” app’y
operation if bowel viability is indeterminate ––––––– INFLAMMATORY BOWEL DISEASE –––––––
Appendicitis in kids: children with appendicitis
SBO × pneumoperitoneum: indicates perforation Inflammatory bowel disease: Crohn’s disease and present more often with a ruptured appendix
due to ischemic or overdistended bowel; ulcerative colitis; presents as crampy abdominal pain,
indicated for ex lap → dead bowel resection bloody diarrhea, and recent weight loss Appendicitis in elderly: older pts usually don’t
have classic presentation of appendicitis, but
Nicked bowel during LOA: small hole → primary IBD management: colonoscopy to determine if rather vague abdominal complaints, sepsis,
repair, large or multiple holes → bowel resection; it’s UC, Crohn’s, or something else → abdominal altered MS, or failure to thrive
high risk of leakage and EC fistula formation CT scan for confirmation of IBD
Appendicitis in pregnancy: enlarged uterus can
Uncertain about SBO: get an upper GI series w/ small Crohn’s disease: inflammatory disease involving full push appendix upwards → RUQ pain; appy can
bowel follow-through, barium contrast will stop at GI tract with skip lesions (terminal ileum is MC site), be performed safely w/o risk to mom or child
site of obstruction if SBO exists Bx shows full thickness + noncaseating granulomas +
creeping fat, Dx terminal ileum string sign on CT scan RLQ pain × dysuria: indicates appendicits, UTI, or
Mesenteric ischemia: presents as postprandial abd appendiceal abscess next to bladder
pain, weight loss, SBO, and multiple abd bruits Crohn’s management: IV steroids and 5-ASA
usually due to atherosclerosis of celiac trunk or SMA (sulfsalazine) for acute flare-ups RLQ pain × minimal dysuria: likely appendicitis

Mesenteric ischemia management: Dx Crohn’s × SBO: due to stenotic terminal ileum; RLQ pain × hematuria: indicates severe UTI or a
mesenteric angiogram, Tx revascularization; manage with NPO, TPN, and observation → if it kidney stone
follow-up with aspirin and evaluation for other fails to resolve, surgical stricturoplasty indicated
atherosclerotic diseases RLQ pain × h/o PID: could be appendicitis, recurrent
Crohn’s × perianal disease: metronidazole PID, or an ectopic; PID confirmed by cervical or
adnexal tenderness, or cervical discharge (get gyn
consult and stain discharge)
Surgery – Lower GI Surgery

RLQ pain × h/o gastroenteritis: could be appendicitis Diverticulitis complications: abscess (Tx perc drain), FAP pts should always get upper endoscopy and
but likely gastroenteritis obstructions, fistula (pneumaturia or fecaluria, Tx remove the duodenal polyps
surgical separation of colon from bladder)
RLQ pain × R pelvic tenderness: likely retrocecal HNPCC: ∆MLH or ∆MSH mismatch repair genes →
appendicitis, go to the OR ––––––––––– LARGE BOWEL DISEASES –––––––––– microsatellite instability, Tx total abdominal
colectomy + ileorectal anastomosis
RLQ pain × BPH Sx: likely bladder outlet obstruction Pseudomembranous colitis: watery diarrhea and
due to enlarged prostate, Tx Foley catheter ↑↑WBC due to clindamycin-related C. diff overgrowth; Colon cancer recurrence: f/u with colonoscopy, CEA
Dx stool toxin or colonoscopy, Tx stop clindamycin marker, CXR for lung mets, and LFTs for liver mets
RLQ pain × h/o IBD: IBD can present similarly to and start metronidazole or vancomycin
appendicitis, get colonoscopy and CT scan and –––––––––––– ANORECTAL DISEASES –––––––––––
manage appropriately Colon cancer screening: anyone above 50 y/o should
get yearly fecal occult blood test (flex sig and All anorectal diseases: first step in management is to
RLQ pain × crampy pain/diarrhea: indicates IBD, colonoscopy also used, but less commonly) scope and rule out cancer
constipation, or cancer rather than appendicitis
Polyps: tubular/pedunculated have a stalk, Hemorrhoids: presents as blood streaks in stool and
RLQ pain × corticosteroids: steroids can mask all Sx sessile/villous are flattened; progression from polyp extreme pain (external); Tx scope to r/o cancer →
of inflammation, be cautious since most steroid pts to cancer takes ~10 years fiber/stool softeners → if it keeps bleeding, excision
won’t present until perforation occurs or banding
Pedunculated polyp: polypectomy w/ biopsy →
App’y findings: inflamed appendix, perforated if dysplasia is not localized to head of polyp, then Non-healing hemorrhoids: indicates anal canal
appendix, normal appendix, inflamed cecum, segmental colectomy cancer; Tx chemoradiation (5-FU) → APR if it
fecalith, carcinoid tumor, other tumors persists or recurs
Sessile polyp: biopsy → if any high grade
Inflamed appendix: take it out dysplasia, then colectomy LAR vs. APR: LAR is low anterior resection (high in
rectum), APR is abdominoperineal resection (low in
Normal appendix: take it out anyways unless Benign polyps: juvenile, Peutz-Jeghers, rectum)
cecum is inflamed, check other areas for inflammatory, hyperplastic; leave these alone
abnormalities (terminal ileum for Crohn’s or Rectal cancer: Dx colonoscopy to r/o cancer, then
Meckel’s, perforated gastric or duodenal ulcer, Colon cancer: left side obstructs, right side bleeds; endoscopic U/S for staging → Tx depends on stage
ovaries, etc.) Dx “apple core” lesion on barium enema, 99%
garden variety (∆MCC, ∆DCC, ∆RAS, ∆p53) and 1% Rectal cancer T1: transanal resection
Inflamed appendix/cecum: take them both out familial inheritance (FAP, HNPCC)
via right colectomy Rectal cancer >T1: neoadjuvant chemoradiation
Stage I/II: localized cancer, Tx colectomy for 6 weeks to shrink the tumor → then LAR with
Inflamed appendix + fecalith: confirms 2cm distal margins → APR if margins not possible
appendicitis Stage III: LN involvement, Tx colectomy and
chemo (5-FU + levimasole, only stage that chemo Anal cancer: presentation is non-specific (bleeding,
Perforated appendix: take it out, I/D the abscess, will improve survival rates) pain, drainage, itching) and requires biopsy
and leave a drain in to prevent wound infection
Stage IV: distal metastasis, Tx palliative chemo Small anal cancer: Tx resection w/ negative
Appendiceal carcinoid: <2cm appendectomy, unless there is a resectable mass in liver or lung margins
>2cm right hemicolectomy; can present w/
carcinoid syndrome due to liver metastasis Colectomy: requires 12-14 LN removal for adequate Large anal cancer: often mistaken as “non-
procedure; requires pre-op bowel prep and abx healing hemorrhoids”, Tx chemoradiation (Nigro
Terminal ileal carcinoid: resection and check the protocol) → APR if it persists or recurs
rest of bowel for other carcinoids; can present Post-op wound infection: open fascia to make
w/ carcinoid syndrome due to liver metastasis sure it’s intact, then local wound care Perirectal abscess: Tx I&D → 40% will develop a
fistula according to Goodsall’s rule (anterior straight
––––––––––––––– DIVERTICULITIS ––––––––––––––– Post-op feculent leak: indicates anastomotic leak, tract, posterior curved tract) → Tx fistulotomy
get abd CT scan to check for undrained collection,
Diverticulosis: presents as LGIB, 85% stop then NPO/IVF is sufficient for most cases Pilonidal abscess: abscess in sacrococcygeal area of
spontaneously, may develop into diverticulitis lower back, Tx I&D and removal of hair
Post-op feculent vomit: indicates either post-op
Diverticulitis: presents like left-sided appendicitis, ileus (due to leakage) or mechanical obstruction; Anal fissure: presents as pain and blood with BM
get CT scan and manage non-op if possible (IVF, NPO, Tx NPO/IVF and NG tube due to increased sphincter tone; Tx fiber/stool
IV antibiotics) even with signs of LLQ peritonitis softeners → if it won’t heal, botox → still won’t heal,
Post-op abscess: Dx CT scan, Tx perc drain lateral internal sphincterotomy
Diverticulitis f/u: colonoscopy to confirm
presence of diverticula and absence of cancer Post-op constipation: indicates either stricture or Recurring anal fissure: probably Crohn’s; biopsy
cancer recurrence; Dx colonoscopy first for possible cancer, then Tx metronidazole if
Surgery indications: uncontained perforation biopsy reveals Crohn’s
that presents as 4-quadrant peritonitis or FAP: ∆APC on chromosome 5p, autosomal dominant,
pneumoperitoneum (air under diaphragm) presents as >100 polyps in colon; Tx either total Rectal prolapse: presents as rectal protrusion
proctocolectomy, or total abdominal colectomy + following defecation; internal prolapse → high-fiber
Elective surgery indications: recurrent episodes strip anal mucosa + ileoanal anastomosis diet to normalize BM, external/bleeding prolapse →
(2x in 6 months, 3x in a year) rectopexy or LAR
Gardner syndrome: FAP + polyps in stomach (not
premalignant) and duodenum (premalignant);
Surgery – Lower GI Surgery

–––––––––––––––– GI BLEEDING ––––––––––––––––

Hematemesis: bloody vomiting, always UGIB

Melena: coffee ground blood in stool, usually UGIB


but can also be from ascending colon

Hematochezia: BRBPR, can be either UGIB or LGIB;


NG tube aspirate/lavage → LGIB if –blood/+bile,
UGIB if +blood, indeterminate if –blood/–bile

UGIB: GI bleeding proximal to Ligament of Treitz,


management is EGD

UGIB in ICU pt: most likely a stress ulcer

LGIB: GI bleeding distal to Ligament of Treitz, 85%


stop spontaneously; MCC are AVM, diverticulosis,
and colon cancer

LGIB management: must localize site of bleeding


w/ tagged RBCs (<1 cc/min) or angiography (>1
cc/min); if not currently bleeding, get upper and
lower endoscopy since localization won’t help

LGIB surgery indications: “hemodynamic


instability despite transfusion”, loss of 4-6 units in
24 hrs or 8-10 units in 48 hrs

LGIB scenario: GI bleed stops spontaneously →


get colonoscopy to determine cause and rule out
possibility of colon cancer → coagulate if AVM,
leave alone if diverticulosis

LGIB scenario: unstable patient, massive BRBPR,


no imaging to localize site of bleeding → total
colectomy since 85% of bleeding is from the colon

LGIB in kids: indicates Meckel’s diverticulum, Dx


technetium uptake into ectopic gastric mucosa

Dx tagged RBCs: sensitive for LGIB at 0.1 cc/min, but


not specific for localizing site of LGIB; bladder always
lights up first so ignore that part

Dx angiogram: less sensitive for LGIB at 1.0 cc/min,


but more specific for localizing site of LGIB; allows
for smaller LGI resection in surgery if necessary

––––––––––––– OTHER GI DISEASES ––––––––––––

Sigmoid volvulus: twisting of sigmoid colon around


mesentery results in closed-loop obstruction; Tx
“detorse” the colon via rigid scope and rectal tube,
then elective sigmoid colectomy

Cecal volvulus: twisting of cecum; Tx right


colectomy since detorsion usually won’t work

Ogilvie’s syndrome: pseudoobstruction and massive


colon dilation w/o mechanical obstruction; Tx
endoscopic decompression or neostigmine if >11 cm
due to possibility of cecal perforation

Constipation: do a rectal exam to make sure stool


isn’t impacted, then give enemas from below
Surgery – Endocrine Surgery

–––––––––––––– THYROID DISEASES––––––––––––– Papillary cancer Tx: total thyroidectomy w/ DiGeorge syndrome: congenital absence of both 3rd
central LN excision → modified radical neck and 4th branchial pouches → hypocalcemia, thymic
Thyroid embryology: derivative of foramen cecum dissection if LN are positive for cancer aplasia
from the base of the tongue
Follicular cancer: endemic to iodine-deficient PTH: secreted from chief cells; three functions –
Thyroglossal duct cyst: remnant of thyroglossal regions, hematogenous spread, 2nd best Px renal ↑Ca++/↓P, bone ↑Ca++/↑P, stimulates 1α-
duct as a mobile midline neck mass, Dx at 1-2 yo hydroxylase to activate vitamin D for indirect GI
due to neck fat, Tx Sistrunk operation (take out Follicular cancer Tx: hemilobectomy with biopsy resorption of Ca++
cyst, trunk, and medial portion of hyoid bone) because 80% are benign adenomas and 20% are
malignant carcinomas, total thyroidectomy if Bx Osteitis fibrosa cystica: rapid, painful loss of bone
Thyroid nodule management: first get TSH levels → reveals malignant carcinoma due to HPTH; “moth eaten skull”
if euthyroid, get FNA for cancer Dx; otherwise, work
up for hypo- or hyperthyroidism Medullary cancer: AD inheritance with MEN2A/2B Calciphylaxis: calcium deposition in soft tissue due
syndromes, Δret proto-oncogene, both lymphatic to HPTH
Risk of malignancy: solid nodules, cold nodules and hematogenous spread, Bx amyloid deposits,
(lack of radioiodine uptake), size > 1.5 cm 3rd best Px Multiple myeloma: punched-out lesions of bone due
to a B-cell neoplasm
Colloid nodule: benign nodule, manage medically Medullary cancer Tx: total thyroidectomy for
both cancer and ret+ prophylaxis w/ central LN Hypercalcemia Sx: kidney stones, painful bones,
Hyperthyroidism: Graves’ disease (most common), excision → modified radical neck dissection if LN abdominal groans (peptic ulcers, pancreatitis,
toxic adenoma (#2), toxic multinodular goiter (#3) are positive for cancer cholelithiasis), psychic overtones (↓mental status)

Graves’ disease (diffuse toxic goiter): auto- Anaplastic cancer: rare, worst Px Hypercalcemia DDx: parathyroid adenoma (MCC
immune disease due to IgG against TSH-R causing outpatient), metastases to bone (MCC inpatient)
exophthalmos, pretibial myxedema, and Anaplastic cancer Tx: palliative (e.g.
hyperthyroidism; more common in women tracheostomy) 1° HPTH: ↑PTH/↑Ca++/↓P, most sensitive test is Cl:P
ratio >33:1; causes include 90% adenoma, 9% hyper-
Graves’ disease Tx: oral radioiodine (no surgical Thyroid lymphoma: associated w/ chronic plasia, 1% carcinoma
risks) or surgery (won’t become hypothyroid) Hashimoto’s thyroiditis, Tx radiation therapy
1° HPTH management: straight to surgery if
Thyroid storm Tx: β-blockers, antithyroid drugs Post-thyroidectomy f/u: take patient off T4 for 6 symptomatic (4-gland exploration or minimally
weeks (use T3 instead for its shorter T½), increase invasive surgery), no imaging necessary
Antithyroid drugs: propylthiouracil (PTU) and TSH level, and make sure total thyroidectomy was
methimazole block peroxidase, PTU also blocks performed; give radioactive iodine exam after 6 4-gland exploration: open exploration of neck
peripheral conversion of T4 to T3 weeks to check if any metastases will light up area; adenoma → take it out; hyperplasia → take
3½ glands out, or take all 4 out and auto-
Toxic adenoma: Dx hot nodule on scan with Positive for metastases: get CT scan, macro → transplant ½ gland back in; carcinoma → en bloc
peripheral suppression, Tx lobectomy if >2 cm, excision, micro → oral radio-iodine therapy resection of parathyroid and ipsi thyroid lobe
radio-iodine if <2 cm
Papillary cancer f/u: follow thyroglobulin levels Minimally invasive surgery: needs positive
Toxic multinodular goiter: Tx surgical excision if Sestamibi scan pre-op, big/abnormal gland during
>2cm, radio-iodine if <2 cm Follicular cancer f/u: follow thyroglobulin levels exploration, and intra-operative decrease in PTH
levels by >50% following excision
Hypothyroidism: chronic/Hashimoto thyroiditis Medullary cancer f/u: 24-hour urine for VMA or
(most common), subacute/De Quervain thyroiditis, metanephrines (MEN syndrome can present with 2° HPTH: ↑↑PTH/↓Ca++/↑P, due to renal failure
acute bacterial thyroiditis pheochromocytoma), follow calcitonin levels
2° HPTH management: medical management, 3½
Hashimoto thyroiditis: autoimmune disease with Modified radical neck dissection: like radical neck gland excision if complications exist
lymphocytic infiltration, Tx Synthroid dissection, but spares sternocleidomastoid muscle,
CN XI, and internal jugular vein 3° HPTH: ↑PTH/↑Ca++/↓P, found in renal transplant
Hashimoto thyroiditis antibodies: anti-TSH, pts whose parathyroid glands work autonomously
anti-microsomal, anti-thyroglobulin, anti- Thyroid cancer Px: MACIS system – Metastasis (non-
peroxidase lymph nodal), Age (<45 is better), Completeness of 3° HPTH management: 3½ gland excision if HPTH
resection, Invasion, Size (>2 cm has worse Px) persists for 1 year following transplant
De Quervain thyroiditis: subacute, painful
granulomatous inflammation following viral URI, Thyroid surgery risk factors: superior laryngeal Sestamibi scan: “lighting up” in parathyroid gland is
Tx observation (condition will self-resolve) nerve (soft/deep voice), recurrent laryngeal nerve 85% accurate for Dx parathyroid adenoma
(hoarseness unilateral, dyspnea bilateral), para-
Acute thyroiditis: painful, swollen, tender thyroid thyroid glands (hypoparathyroidism) Hypercalcemic crisis: Ca++>15 indicating carcinoma;
mass due to Staph/Strep infection, Tx I&D Tx flush with NS, then drain with furosemide (Lasix),
––––––––––––– PARATHYROID DISEASES ––––––––– then treat the underlying cause
Thyroid cancers: 80% papillary, 10% follicular, 4%
medullary, 1% anaplastic; FNA can be used to Dx Parathyroid glands: 30-50 mg each, brownish-yellow Parathyroid nodule: probably a thyroid nodule,
papillary and medullary but not follicular (must see color, sinks in water (to differentiate from fat) unless Ca++ ~15 indicating parathyroid carcinoma
capsular invasion to DDx adenoma vs. carcinoma)
Parathyroid embryology: 3rd branchial pouch 2nd operation s/p adenoma removal: requires
Papillary cancer: associated with radiation, becomes inferior glands and thymus, 4th becomes localization studies, don’t just go back in there
lymphatic spread, best Px overall, Bx shows superior glands
Orphan Annie nuclei with psamomma bodies
Surgery – Endocrine Surgery

Missing parathyroid glands: check thymus (MC Pituitary Cushing: due to ↑ACTH secretion,
location/15%), up and down neck, open carotid suppressable by high dose dexamethasone;
sheath, retro-esophageal area, esophageal groove, Dx MRI, Tx transsphenoid hypophysectomy
and inside the thyroid gland itself
Adrenal Cushing: due to ↑cortisol secretion
Median sternotomy: only indicated if pt is in a life- (causes ↓ACTH); Dx CT scan, Tx adrenalectomy
threatening hypercalcemic crisis
Ectopic Cushing: due to ↑ACTH secretion usually
–––––– MULTIPLE ENDOCRINE NEOPLASIA –––––– from SCLC, not suppressable by high dose dexa

MEN1 (Wermer): pituitary adenoma, pancreatic Adrenal gland tumors: aldosteronoma, cortisoloma,
endocrine cancer (MC gastrinoma), parathyroid sex hormone-secreting tumor, pheochromocytoma,
hyperplasia incidentaloma

MEN1 Tx: excise parathyroid first since hyper- Aldosteronoma: presents as ↑Na, ↓K, ↓H, ↓renin;
calcemia can cause ↑gastrin levels; if Z-E can’t be Dx CT scan for mass (if no mass present or ↑renin,
controlled w/ PPIs → total gastrectomy since think RA stenosis), Tx adrenalectomy (adenoma)
MEN gastrinomas are multifocal or aldactone (hyperplasia)

MEN2A (Sipple): parathyroid hyperplasia, Cortisoloma: presents as adrenal Cushing’s, Tx


pheochromocytoma, thyroid medullary cancer adrenalectomy

MEN2B: pheochromocytoma, thyroid medullary Sex hormone-secreting tumor: Tx adrenalectomy


cancer, mucosal neuromas, Marfanoid habitus
Pheochromocytoma: presents with HA, HTN,
MEN2A/2B Tx: excise pheochromocytoma first palpitations, etc.; Dx 24 hour urinary VMA or
since it can be life-threatening metanephrines, Tx α-blockers (phenoxybenz-
amine) then β-blockers then adrenalectomy
Gastrinoma: give PPIs → if acid persists, get serum
gastrin levels → <200 absent, >500 present, 200-500 Intraoperative hypotension: give IVF
indeterminate and requires secretin stimulation test
→ if paradoxical ↑gastrin w/ secretin, present → get Incidentaloma: make sure it’s not secreting
CT scan to localize and cut it out hormones then work-up based on size; <5 cm →
leave alone, >5 cm → resection + check other organs
VIPoma: presents as “rice water” diarrhea; get CT since adrenals are common site of metastasis
scan to localize and cut it out
Aortic coarctation: presents as HTN in arms but
Insulinoma: presents as hypoglycemia, Dx ↑C- normal BP in legs; get CXR (scalloping of ribs) then
peptide levels or monitored fasting (to see if they’re confirm w/ spiral CT, then surgical correction
“faking it”); get CT scan and cut it out
Renovascular HTN: presents as HTN + abd bruits,
Glucagonoma: presents as new-onset diabetes and secondary to fibromuscular dysplasia (young women)
migratory rash from lower body upwards, Dx or atherosclerosis (old men) of the renal arteries;
glucose challenge test; get CT scan and cut it out get a doppler of renal vessels, then either balloon
dilation or surgical correction
Metastatic glucagonoma: give somatostatin and
streptozocin since surgery is contraindicated

Nesidioblastosis: presents as hypersecretion of


insulin in a newborn; Tx 95% pancreatectomy

–––––––––– OTHER ENDOCRINE DISEASES –––––––––

Pituitary adenoma: prolactinoma (#1), null cell tumor


(#2), ACTH (#3), GH (#4); may present as bitemporal
hemianopsia ◐◑ due to optic chiasm compression

Prolactinoma: presents with galactorrhea and


amenorrhea, Tx bromocriptine

Null cell tumor: doesn’t secrete anything

Cushing syndrome: presents as truncal obesity, abd


striae, buffalo hump, hyperglycemia, osteoporosis,
HTN, and immunosuppression due to ↑cortisol

Cushing causes: MCC is exogenous steroid use;


intrinsic causes include pituitary adenoma,
adrenal adenoma, and ectopic secretion of ACTH
Surgery – Skin/Soft Tissue Surgery

––––––––––––––– SKIN CANCERS ––––––––––––––– –––––––––––––––– SARCOMAS –––––––––––––––– Hernia surgery types: open (Bassini, McVay,
Shouldice, Lichtenstein), laparoscopic (TAPP, TEP)
Skin cancers: basal cell carcinoma (50%), squamous Sarcomas: present as firm, painless masses; Px
cell carcinoma (25%), malignant melanoma (15%); depends on size, grade, and distant metastases (no Bassini repair: inguinal ligament to conjoint
management is full-thickness incisional biopsy at LN since sarcomas spread hematogenously) tendon, high tension, fixes direct and indirect
border of the lesion hernias
Sarcoma management: Dx incisional biopsy
Basal cell carcinoma: raised waxy lesion or non- parallel to resection margins, chest CT for mets, McVay repair: Cooper’s ligament to conjoint
healing ulcer in upper face, doesn’t spread; Tx local Tx resection w/ 1 cm negative margins tendon; fixes direct, indirect, and femoral hernias
excision w/ 1 mm margins
High-grade sarcomas: Tx radical amputation w/ McVay repair w/ relaxing incision: cut
Squamous cell carcinoma: non-healing ulcer in lower post-op radiation therapy transversus abdominis aponeurosis to relax
face, spreads to LN; Tx local excision w/ 1 cm margins tension on sutures
and LN excision Sarcoma × lung mets: get chest CT to characterize
lesion and look for others → needle biopsy for Shouldice repair: inguinal ligament to reinforced
SCC risk factors: draining fistulas, arsenic confirmation → Tx thoracic wedge resection transversalis fascia in two layers
exposure
Sarcoma × liver mets: biopsy for confirmation → Tx Lichtenstein repair: tension-free repair w/ mesh;
Keratoacanthoma: variant of SCC that grows hepatic wedge resection or formal lobectomy fixes direct, indirect, and femoral hernias
rapidly and then dies off spontaneously
––––––––––––––––– HERNIAS ––––––––––––––––– TAPP and TEP: TransAbdominal PrePeritoneal
Dysplastic nevus: atypical mole, precursor of and Totally ExtraPeritoneal lap procedures;
malignant melanomas, requires close observation Hernia surgery indications: all abdominal hernias have steep learning curve and unproven results
due to possibility of strangulation, except sliding
Melanoma: ABCDE – Asymmetric, irregular Borders, esophageal hernias (Tx PPIs) and umbilical hernias in Hernia surgery complications: damage to genital
different Colors, Diameter >5mm, Evolving; Px pts <2 yo (simple observation) branch of genitofemoral nerve, ilioinguinal,
related to depth, highly metastatic and will be found iliohypogastric, or lateral femoral cutaneous nerves
in weird places Umbilical hernia management: <2cm watch, causing pain and sensory defects
>2cm fix, fix if it doesn’t regress before kinder-
Ulcerated melanoma: 1/3 reduction in survival garten regardless Hernia surgery post-op: avoid lifting for 6 weeks so
incision site has time to regain strength
Depth <0.75 mm: Tx excision w/ 1 cm margin, Hernia × SBO: presents as abdominal distention,
good prognosis nausea, and vomiting due to incarcerated or
strangulated hernia; Tx emergent hernia repair
Depth 0.76-3.99 mm: Tx excision w/ 2 cm margin,
palpable LN → removal, no palpable LN → Hernia × strangulation: presents as firm/tender
sentinel LN biopsy w/ elective removal mass w/ fever, ↑WBC, metabolic acidosis; Tx
emergent hernia repair
Depth >4 mm: Tx excision w/ 2 cm margin,
remove palpable LN, pt will likely die from mets Indirect hernia: through internal ring lateral to
inferior epigastrics vessels, due to patent processus
Melanoma × LN involvement: remove regional LN, vaginalis; most common hernia overall (even in
then complete staging for mets (CXR, CT scan, LFTs, women and elderly)
brain MRI)
Indirect hernia in kids: high incidence of
Melanoma × metastases: Tx radiation and bilaterality, repair is limited to high ligation of sac
chemotherapy (interferons) w/o abdominal wall repair

Lentigo maligna melanoma: superficial, spreading Direct hernia: medial to inferior epigastric vessels,
melanoma on the face, good prognosis overall, Tx due to weakening abdominal wall; more common in
excision w/ narrow margin elderly

Acral lentiginous melanoma: melanoma on pale Femoral hernia: below inguinal ligament into
areas of dark-skinned pts (sole of feet, palm of hand), femoral triangle, more common in women, highest
worst prognosis due to depth risk of strangulation (50% of all strangulations)

Subungual melanoma: melanoma under fingernail or Sliding hernia: involves other viscera as part of the
toenail, Tx DIP amputation hernia wall (e.g. bladder, cecum, sigmoid colon),
important to recognize sliding hernias as not to
Melanoma on face: Tx excision w/ smaller margins injure any contained structures
for cosmetic purposes, consult plastic surgery
Ventral hernia: hernia through incision site at linea
Melanoma in anus: all mucosal melanomas have bad alba; Tx primary closure (small) or mesh repair (large)
prognosis, Tx APR w/ palpable LN excision
Rare hernia causes: anything that increases
Melanoma × SBO: presents as abd distention, abdominal pressure – obesity, COPD, ascites, BPH
nausea and vomiting due to metastatic melanoma in causing bladder obstruction, constipation, ascites;
peritoneal cavity; Tx ex lap w/ excision these require further evaluation before surgery
Surgery – Breast Surgery

–––––––––––– BREAST MANAGEMENT ––––––––––– –––––––– MALIGNANT BREAST DISEASES –––––––

Breast cancer screening: monthly self-exam, yearly Breast cancer buzzwords: family history, ill-defined
breast exam, yearly mammogram if >40 y/o (earlier if or fixed mass, skin or nipple retraction, peau
there is a strong family Hx) d’orange, nipple eczema, and palpable axillary LN

Breast mass on exam: first, get mammogram to Breast cancer Tx: identical for all cancers (except
characterize mass and rule out other masses → inflammatory, LCIS, DCIS); depends on cancer stage
get core needle biopsy → excisional biopsy if
needle inadequate Stage I-II: lumpectomy w/ 1 cm negative margins
+ axillary LN samping + post-op radiation; or
Abnormality on mammogram: ‘probably benign’ modified radical mastectomy
→ follow-up; ‘suspicious’ → core needle biopsy,
excisional biopsy if needle inadequate Stage III-IV: systemic chemotherapy

Breast cancer risk factors: family Hx (#1), BRCA1 and Axillary LN involvement: Tx chemotherapy if pre-
BRCA2 genes, old age menopausal, tamoxifen if post-menopausal

MC breast masses: <25 fibroadenoma, 25-50 fibro- Breast cancer × pregnancy: work-up and Tx is
cystic change, >50 infiltrating ductal carcinoma identical, except no chemo during 1st trimester
and no radiation anytime; if stage III or IV,
–––––––––– BENIGN BREAST DISEASES ––––––––– chemoradiation is essential and abortion may be
necessary
Fibroadenoma: firm, mobile, rubbery mass in young
women; Dx FNA and U/S, Tx elective removal Breast cancer × men: work-up and Tx is identical

Giant juvenile fibroadenoma: seen in teens with Paget’s disease: presents as “crusty” nipple eczema,
rapid breast growth; Tx excision to avoid breast indicates underlying infiltrating ductal carcinoma
deformity
Inflammatory cancer: resembles cellulitis w/ peau
Phyllodes tumor: huge, benign tumors that distort d’orange, “tumor cells in dermal lymphatics”, Tx
the breast; Dx core or incisional biopsy since FNA is requires pre-op chemo
insufficient, Tx excision w/ negative margins
DCIS: Tx simple mastectomy
Fibrocystic change: multiple, bilateral lumps that
vary with menstrual cycle; if dominant or persistent LCIS: Tx careful observation or prophylactic bilateral
lump → aspiration → excisional biopsy if mass simple mastectomy; not precancerous, but is a risk
recurs or persists factor of breast cancers

Sclerosing adenosis: Tx careful observation or –––––––– POST-SURGERY COMPLICATIONS –––––––


prophylactic bilateral simple mastectomy; not
precancerous, but is a risk factor of cancers Small ipsilateral nodule: local recurrence until
proven otherwise, work-up like any other mass
Atypical ductal hyperplasia: Tx excision; not
precancerous, but high risk of becoming a cancer Contralateral finding: most likely a new primary
cancer, work-up like any other mass
Intraductal papilloma: presents as bloody nipple
discharge; Tx galactogram/ductogram-guided Elevated LFTs: indicates liver mets, get abd CT scan
excision (due to small risk of carcinoma)
Back pain: indicates bone mets, get bone scan then
Acute mastitis: breastfeeding women get X-ray
Staph/Strep infection through cracks in nipple w/
yellowish discharge, Tx antibiotics and warm Pathologic fracture: indicates bone mets, get bone
compresses, may develop into breast abscess scan then X-ray

Breast abscess: pus pocket in lactating women; Sensorimotor defects: indicates spine mets, get MRI
Tx I&D and biopsy to rule out possible cancer
Severe headache: indicates brain mets, get MRI
Periductal mastitis: subareolar infection in smokers,
both males and females affected New-onset seizures: indicates brain mets, get MRI

Gynecomastia: breast tissue hypertrophy in males; New-onset coma: indicates acute hypercalcemia due
common in puberty (will resolve spontaneously) and to bone mets, get a renal panel for confirmation
older men (excision if it doesn’t regress on its own)

Dystrophic calcification: presents as breast lump


following trauma, but work up as a cancer until
proven otherwise (trick question!)
Surgery – Acute/Critical Care Surgery

–––––––––––– TRAUMA EVALUATION –––––––––––– C-spine injury: suspected with neurological Sx, Stab × hemiparesis: suggests injury to the carotid
radiological abnormalities, or C-spine tenderness; artery; get angiogram and either vascular or
Evaluation of trauma pt: primary survey, get an intubation requires extreme caution neurosurg consult
“ample” Hx, secondary survey
Priapism: indicates fresh spinal cord injury; check Neck GSW: exploration of neck is mandatory in zone
Primary survey: initial evaluation of trauma pt; for anal sphincter tone, bradycardia, and possibly 2, observe if stable in zones 1+3
ABCDE – airway, breathing, circulation, disability neurogenic shock
(neurological), environment/exposure Neck blunt trauma: at risk of carotid dissection (Tx
––––––––––––––– HEAD TRAUMA ––––––––––––––– anticoagulation) or laryngeal edema (Tx intubation)
Ample Hx: things to ask a trauma pt – allergies,
meds, previous illnesses, last meal, events Head trauma evaluation: begin with primary survey ––––––––––––––– SPINAL TRAUMA ––––––––––––––
surrounding injury (ABCs) → rapid neuro exam if stable → if any neuro
sx, get head CT and neurosurg consult Spinal cord injury management: get MRI to look at
Secondary survey: full inspection of trauma pt spinal cord, then immediate high-dose steroids
following clearance of life-threatening issues, Rapid neuro exam: check pupils, cranial nerves,
continual reassessment is necessary for changes peripheral motor/sensory function, and rate level of Hemisection syndrome: always due to stab
consciousness via GCS wounds in posterior neck area, presents as ipsi
Airway: determine if airway is clear, quickest way to DCML/motor loss and contra ACL loss
evaluate is to see if pt can talk; use intubation, EDH: head CT shows convex lens hematoma;
cricothyroidotomy, or tracheostomy if obstructed presents as LOC → lucid interval → LOC → Anterior cord syndrome: usually seen w/
ipsilateral fixed/dilated pupil, Tx craniotomy vertebral burst fractures, presents as bilateral
Blunt trauma to neck: consider possibility of ALS/motor loss but intact DCML
laryngeal edema developing into airway SDH: head CT shows crescent moon hematoma,
obstruction high risk for brain herniation; Tx head elevation, Central cord syndrome: usually seen w/ whiplash
hyperventilate, sedate, mannitol + furosemide (e.g. rear end collisions), presents as UE burning
Breathing: evaluate via lung auscultation, CXR, and pain and paralysis, but LE nerves intact
pulse oximetry Diffuse axonal injury: head CT shows blurred
gray-white junction and small punctate ––––––––––––––– CHEST TRAUMA ––––––––––––––
Circulation: stop external hemorrhage, 2 large-bore hemorrhages; management is prevention of ↑ICP
IV lines (14 or 16-gauge) w/ 2L crystalloid, followed Rib fx: painful breathing leads to shallow breaths →
by packed RBC (type O–) if necessary Head trauma × hypovolemic shock: look for another atelectasis → pneumonia, Tx local nerve block
source of bleeding
Cardiac tamponade: presents with Beck’s triad – Chest stab wounds: suspect HTX or PTX, chest tube
JVD, muffled heart sounds, hypotension; Basal skull fx: raccoon eyes, hemotympanum, insertion is indicated
Tx subxiphoid pericardiocentesis and go to OR otorrhea, rhinorrhea, ecchymosis behind the ear
(Battle’s sign); neurosurg consult is indicated Infraclavicular stab wounds: suspect injury to
Hypovolemic shock: class I <15% and lactic acid; subclavicular artery or vein, Dx angiogram if pt is
class II 15-30% and ↑HR; class III 30-40% and ↓BP; ––––––––––––––– NECK TRAUMA ––––––––––––––– stable or urgent exploration if unstable
class IV >40% with lethargy and anuria
Neck stab wound: begin with primary survey (ABCs), Nipple-level stab wounds: suspect additional
Hypovolemic shock Tx: 2 large-bore IV lines and then careful exam for injured structures injury to diaphragm/abdominal organs, ex lap
2L crystalloid infusion, resuscitation measured by indicated for abdominal organ damage
urine output, HR, BP, and mental status Surgery indications: expanding hematoma,
deteriorating vital signs, coughing blood, PTX: air in pleural space; insert a finger to make sure
Hypovolemic shock Tx unresponsive: search for subcutaneous emphysema, all GSW and stab of pleural space, then insert a chest tube, remove
underlying cause, ex lap or thoracotomy may be wounds in zone 2 (zones 1+3 can be observed if when lung is fully inflated and no air leak present
indicated for continuous internal bleeding stable)
Small PTX: simple observation is okay if it’s
Closed head injury: brain edema and ischemia Stab × zone 1: below cricoid (includes lung); asymptomatic, no free fluid present, not
causes Cushing reflex – peripheral vaso- observation and pre-op angiogram if stable, expanding, and no other significant injuries
constriction (↑BP), bradycardia (↓HR), and otherwise exploration of neck is necessary
respiratory depression (↓RR) Sucking chest wound: seal with occlusive
Stab × zone 2: between mandible and cricoid; dressing, insert chest tube in different location
Circulation × pregnancy: ↑HR is normal and may exploration of neck is mandatory
not indicate shock, ↓hct of 31-35% is normal, best Nonfunctioning chest tube: chest tube is in the
evaluated laying on her left side Stab × zone 3: above mandible; observation and wrong location or not working properly;
pre-op angiogram if stable, otherwise exploration reposition or replace chest tube
Disability: evaluate with GCS; 15 max, ≤8 coma, 3 of neck is necessary
nearly dead Continuous air leak into chest tube: major airway
Stab × dysphagia: suggests injury to the injury with disruption of bronchus or trachea
Disability × awake: stabilize C-spine, palpate esophagus, Dx esophagoscopy or barium
posterior neck for tenderness, assess extremities swallow Tension PTX: mediastinal shift, hypotension, JVD,
for motor/sensory function, and get lateral C- absent breath sounds, and hyperresonant to
spine imaging Stab × hoarseness: suggests injury to the airway, percussion; Tx emergent needle aspiration and
larynx (voicebox), or recurrent laryngeal nerve; chest tube insertion (do not wait for CXR or ABG)
Disability × comatose: C-spine cannot be cleared, Dx laryngoscopy or exploration
so precautions must continue HTX: blood in pleural space, go to OR if chest tube
drains 1.5L or >200 mL/hr for 3 hours
Surgery – Acute/Critical Care Surgery

HTX × hypotension: suspect blood loss in left Trauma × perforated viscera: CXR reveals free air Hemorrhage × abd distention: bleeding into abd
chest, indicated for left chest tube insertion in abdomen; rapid eval of non-abd injuries → cavity can lead to abd compartment syndrome,
repair perforated viscera in the OR which can cause decreased renal blood flow
Thoracic GSW: GSW is managed differently due to (oliguria) and dyspnea (elevated diaphragm)
unpredictable path of bullets vs. knife, mark both Trauma × hypotensive w/o hemorrhage:
entrance/exit sites and get imaging indicated for FAST or DPL → ex lap in OR if ↓CO/↓CVP: hypovolemic shock, neurogenic shock
positive; CT scan inappropriate for unstable pts (e.g. spinal cord trauma, anaphylaxis)
Thoracic blunt trauma: suspect HPTX, chest tube
insertion is indicated; emergent thoracotomy if >1.5L Trauma × hypotensive w/ pelvic fx: suspect ↓CO/↑CVP: cardiogenic shock (e.g. CHF, MI, tension
or >200mL/hr blood is extracted from tube vascular injury from branch of internal iliac, Dx PTX, pericardial tamponade)
FAST, Tx control by embolization
Aortic transection: presents as widened media- ––––––––– RETROPERITONEAL TRAUMA ––––––––
stinum on CXR, confirm w/ aortic angiography or Dx DPL/FAST: diagnostic peritoneal lavage (DPL)
chest CT → go to OR if transected or ultrasound (FAST) useful for quick Dx of Retroperitoneal zones: central is zone 1, flank is
internal free fluid → positive DPL/FAST is zone 2, pelvis is zone 3
Flail chest: presents w/ paradoxic chest wall indicated for ex lap; may be false negative in
movements due to multiple rib fx, suspect lung retroperitoneal injuries Surgery indications: all zone 1 hematomas, zone 2
contusions and aortic transection; manage by and 3 hematomas only if penetrating trauma
inserting bilateral chest tubes and serial ABGs Dx CT scan: useful for stable pts, avoid in
unstable or severely injured pts Zone 1 (central hematoma): may involve injury to
Pulmonary contusions: presents as deteriorating major vascular structure, indicated for ex lap
ABGs and “white out” of lungs on CXR; Tx colloid Splenic laceration: ex lap if unstable, preserve
(not crystalloid) + diuretics + fluid restriction spleen if possible to avoid post-splenectomy sepsis, Zone 2 (flank hematoma): no exploration
avoid blood transfusion if possible, splenectomy warranted unless penetrating trauma, because
Ruptured diaphragm: CXR reveals stomach in left requires vaccination for SHiN bacteria peritoneum will tamponade bleeding
chest; rapid eval of non-abd injuries → go to OR
Liver laceration: ex lap if unstable, observe if stable Zone 3 (pelvic hematoma): no exploration
Ruptured trachea/bronchi: presents as warranted unless penetrating trauma, because
continuous air leak into chest tube and Mesentery injury: difficult to detect on CT so peritoneum will tamponade bleeding
subcutaneous emphysema; go to OR mechanism must evoke suspicion, co-exists with
bowel injury since mesentery is tougher than bowel ––––––––––––– UROLOGIC TRAUMA ––––––––––––
Air embolus: presents as sudden death in an
intubated/respirator pt; management is immediate Renal laceration: stable → angiography and planned Urologic surgery indications: all GSW, stab wounds,
Trendelenburg position + cardiac massage operative repair; unstable → IV pyelo to detect if and other penetrating injuries
two kidneys present, then OR for nephrectomy
Fat embolus: presents as RDS and petechiae in Urethral injury: presents as blood on meatus, scrotal
neck/axilla due to bone marrow embolization from Hematoma in SMA region: suggests major injury to hematoma, and “high-riding” prostate; get a
long bone fx; management is respiratory support abd aorta, major aortic branches, pancreas, or retrograde urethrogram and suprapubic catheter
duodenum; stable → angiography and assessment instead of Foley (may compound injury)
––––––––––––– ABDOMINAL TRAUMA ––––––––––– before operation, unstable → urgent ex lap
Bladder injury: associated w/ seatbelt trauma in
Abdominal GSW: mandatory OR for ex lap for all Pancreatic transection: mandatory ex lap; minor adults; get a retrograde cystogram and post-void
GSW below nipple level injury → debride and drain, major injury → resection films, then surgical repair
of devitalized pancreatic tissue and repair of
Abdominal stab wound: mandatory OR if stab duodenal injury Renal injury: associated w/ blunt trauma to back and
wound penetrates peritoneum (e.g. protruding lower rib fx; get CT scan, manage non-op if possible
viscera, peritonitis, hemodynamic instability); Duodenal hematoma: common in kids hitting abd on
otherwise, digital exploration and observation bicycle handlebars, hematoma causes GI obstruction; Scrotal hematoma: manage non-op unless testicle is
Tx NPO/IVF, will resolve spontaneously in 5-7 days ruptured (seen on U/S)
Abdominal blunt trauma: mandatory OR if signs of
peritonitis or hemodynamic instability despite Blood in ex lap: stop bleeding by packing all four Penis fx: associated w/ cowgirl sex; insert a
transfusion; otherwise, CT scan can be done safely quadrants with gauze packs → attack injuries in suprapubic catheter then repair surgically
before proceeding order of severity and attempt hemostasis → inspect
remainder of abd contents and repair injuries –––––––––––––––– LIMB TRAUMA –––––––––––––––
Trauma × flat/nontender abd: observe if no
mechanism, imaging if mechanism exists Triad of death: acidosis, coagulopathy, hypothermia Penetrating limb trauma: no vascular injury → give
tetanus PPx and clean wound; vascular injury
Trauma × severe abd pain: suspect peritoneal Hemorrhage × hypothermia: can lead to present → arteriogram if stable, OR if unstable
irritation due to blood or intestinal contents, Dx coagulopathy due to platelet dysfunction and
FAST or CT → indicated for ex lap if positive PT/PTT prolongation; rewarming indicated Combined limb trauma: order of repair is bone first,
then vascular repair, nerve last; fasciotomy required
Trauma × tire mark across abd: indicates severe Hemorrhage × low platelets: loss of platelets due to prevent compartment syndrome
direct trauma, high suspicion of internal injury to hemorrhage puts pt at risk for coagulopathy;
platelet transfusion indicated Limb GSW: may require extensive debridements and
Trauma × coma: abd imaging mandatory since amputation if unsalvageable
physical exam is impossible on a comatose pt Hemorrhage × metabolic acidosis: results from
decreased tissue perfusion causing lactic acidosis; Crushing limb trauma: high risk of myoglobinuria
crystalloid infusion indicated leading to acute renal failure; Tx IV fluids + mannitol
+ acetazolamide to maintain a high urine output
Surgery – Acute/Critical Care Surgery

––––––––––––––––––– BURNS –––––––––––––––––– Snake bites: don’t always result in envenomation; if


signs of venom evident (pain, swelling, discoloration)
Burn assessment: determine depth of burn, type of then draw blood for labs and Tx anti-venin
burn, and body surface area (%BSA) burned
Bee stings: may present as anaphylactic shock
First-degree burns: epidermis only, painful (“warm and pink”) secondary to allergic response;
Tx stinger removal and epinephrine
Second-degree burns: extends into dermis,
causes pain and blistering, may develop into Spider bites: black widows have neurotoxins, brown
third-degree burns w/o proper management recluses have necrotoxins

Third-degree burns: full thickness, painless Black widow spiders: present w/ severe muscle
cramps, acute abdomen, and nausea/vomiting;
Burn management: tetanus PPx, IV pain meds, give IV calcium gluconate + muscle relaxants
topical agents (silver sulfadiazine is default, mafenide
acetate for deep penetration, triple abx ointment for Brown recluse spiders: present as an ulcerated
the eyes) lesion; excise ulcer and skin graft

Rule of 9’s: estimates %BSA burned; head and upper Human bites: requires extensive irrigation and
extremities are 9% each; anterior trunk, posterior debridement due to high amounts of bacteria
trunk, and lower extremities are 18% each; and
perineum is the last 1% ––––––––––––––––– NUTRITION ––––––––––––––––

Parkland formula: estimates fluid replacement in TPN: indicated for nutrition when gut is non-
burn victims, LR volume = %BSA × kg × 4mL/kg; functional or not availble, requires personalization of
give 50%LR in first 8 hours, 50%LR in next 16 hours formula for nutrition status

Fluid for infants: 20 mL/kg if BSA>20% TPN × fever: examine catheter site for infection,
obtain cultures from catheter site and blood;
Chemical burns: alkaline burns are worse than acids if –bacteria → change catheter site, if +bacteria
→ change catheter site and start antibiotics
Chemical burn management: massive tap water
irrigation, don’t try acid-base neutralization, TPN × metabolic coma: hyperglycemic,
except in ingestion – orange juice for alkali, milk hyperosmolar, nonketotic coma is commonly due
for acids to excessive osmolar diuresis in hyperglycemia

Electrical burns: may appear benign on surface, but TPN × elevated LFTs: very common (up to 30%),
masks large amounts of interior damage to muscles, TPN can cause fatty liver, structural liver damage,
nerves, and vessels; at risk of cardiac injury and even cirrhosis with prolonged TPN
(arrhythmias) and muscle injury (myoglobinuria)
TPN × dry/scaly skin: indicates free fatty acid
Electrical burn management: IV fluids + mannitol deficiency, Tx FFA administration
+ acetazolamide to maintain a high urine output
Nitrogen in TPN: positive nitrogen balance is the
Inhalation burns: suspected with carbonaceous goal, since negative nitrogen balance indicates a
sputum, facial burns, singed facial/nasal hairs, catabolic state; severity of injury correlates with a
hoarseness, etc. due to smoke inhalation higher amino acid requirement

Inhalation burn management: confirm w/ BEE: basal energy expenditure, equal to (body
fiberoptic bronchoscopy and order serial ABGs; weight × 25 kcal/kg/day)
high COHb levels → Tx 100% O2
Nondepleted pts: 1.2× BEE calories needed; good
Circumferential burns: rapidly become thick and nutritional status before surgery
contracted, causing restricted ventilation in the
chest and ischemia in extremities; Tx escharotomy Depleted pts: 1.2-1.5× BEE calories needed;
malnourished before surgery
Scalding burns in kids: consider child abuse
Hypermetabolic pts: 1.5-2× BEE calories needed;
Burns × methemoglobinemia: Sx chocolate-brown severely stressed catabolic state due to trauma,
blood, central cyanosis of trunk, arrhythmias, burns, sepsis, cancer, etc.
seizures, coma; Dx ABGs (pulse ox is unreliable), Tx
IV methylene blue Macronutrients: carbs (4 kcal/gm), protein (4
kcal/gm), fat (9 kcal/gm), etoh (7 kcal/gm)
––––––––––––––– BITES & STINGS ––––––––––––––

Bite management: all require tetanus PPx

Dog bites: rabies PPx only required if bite was


unprovoked and dog isn’t available for brain Bx
Surgery –Subspecialty Surgery

––––––––––––– PEDIATRIC SURGERY –––––––––––– Vascular rings: presents as stridor, RDS, crowing Embolic occlusion of retinal artery: presents as
respiration, and dysphagia in an infant due to sudden unilateral loss of vision; Tx breathe into
VACTERL syndrome: vertebral, anal atresia, cardiac, compression of trachea and esophagus; Dx barium paper bag and repeatedly press on eye to shake clot
TE fistula, esophageal atresia, renal, limbs (radius); swallow and bronchoscopy, Tx surgical correction into a more distal branch of retinal artery
presence of any of these requires checking for the
others Cardiac anomalies: all require prophylactic anti- ––––––––––––– OTOLARYNGOLOGY ––––––––––––
biotics for subacute bacterial endocarditis
Esophageal atresia: presents as excess salivation, Neck masses: three types – congenital, inflammatory,
choking spells, coiling NG tube, and often a TE L→R shunts: all present with murmurs and late- neoplastic; differentiate via onset and resolution
fistula; Tx surgical repair (if delay is necessary, onset cyanosis due to Eisenmenger syndrome
gastrostomy) (pulmonary HTN reversing direction of shunt) Congenital neck masses: thyroglossal duct cyst
(midline), branchial cleft cyst (lateral), cystic
Anal atresia: high rectal pouch → colostomy then ASD: presents as low-grade systolic murmur, hygroma (base of neck)
delayed repair, low rectal pouch → repair, if ano- fixed S2, and frequent colds; Dx echo, Tx surgery
fistula present → delay repair since further Thyroglossal duct cyst: remnant of thyroglossal
growth may correct condition VSD: presents as pansystolic harsh-sounding duct as a mobile midline neck mass, Dx at 1-2 yo
murmur with failure to thrive; Dx echo, Tx surgery due to neck fat, Tx Sistrunk operation (take out
Congenital diaphragmatic hernia: presents as RDS cyst, trunk, and medial portion of hyoid bone)
due to hypoplastic left lung; Dx CXR shows bowel in PDA: presents with machinery-like murmur, often
left chest, Tx intubation w/ low-pressure ventilation seen with congenital rubella syndrome; Dx echo, Branchial cleft cyst: lateral mass at anterior edge
Tx indomethacin or surgery of SCM, may have a small opening and blind tract
Gastroschisis: midline hernia to left of umbilical cord
tearing through peritoneum; Tx closure if small, silo R→L shunts: all present with murmurs and early- Cystic hygroma: dilated lymphatic duct at base of
if large, and TPN for a month since GI doesn’t work onset cyanosis neck, common in Turner syndrome, get CT scan
to find extent of mass before surgical removal
Omphalocele: midline hernia through the cord with Tetralogy of Fallot: presents as cyanosis and
peritoneal covering; Tx closure if small, silo if large clubbing in a 5 y/o child who squats for relief; LN DDx: cancers are large (>2 cm), painless, fixed,
Px determined by degree of pulmonary stenosis, insidious onset, unusual site; infectious LN are small
Biliary vomiting: indicates SBO distal to ampulla of Dx echo shows RVH, Tx surgery (<2 cm), painful, rapid onset, mobile, normal site
Vater; DDx duodenal atresia, intestinal atresia
annular pancreas, or malrotation Transposition of great vessels: presents as life- Lymphomas: presents with multiple swollen LN,
threatening cyanosis in a newborn; Dx echo, Tx fever, and night sweats; get excisional biopsy, then
Malrotation: presents as biliary vomiting and surgery Tx chemo
double-bubble on X-ray; Dx contrast enema or
upper GI study, Tx emergency surgical correction –––––––––––––– OPHTHALMOLOGY ––––––––––––– Supraclavicular metastases: MC primary sites are
pancreas and stomach; get excisional biopsy for Dx
Intestinal atresia: presents as biliary vomiting and Strabismus: misaligned eyes due to uncoordinated
multiple air-fluid levels on X-ray, aka apple-peel extraocular muscles; Tx surgical extraocular muscle Head/neck SCC: presents as persistent hoarseness,
atresia due to vascular accident in utero manipulation to prevent ambylopia painless ulcer in floor of mouth, or unilateral earache;
risk factors are EtOH, smoking, and AIDS
Necrotizing enterocolitis: presents as feeding Amblyopia: vision impairment usually due to
intolerance, abd distention, and ↓platelets in strabismus in kids, Tx eyepatch the good eye to SCC management: get triple endoscopy to look
premies; Tx NPO, IVF, TPN, IV abx → surgical repair if develop the impaired one for primary tumor → biopsy tumor → CT scan to
signs of intestinal necrosis/perforation present determine stage; Tx resection, radical neck
Retinoblastoma: presents as a leukocoria in a baby, dissection, cisplatin-based chemo
Pyloric stenosis: presents as non-bilious projectile Tx surgical enucleation (remove the eye)
vomiting in first-born boys; Dx palpable epigastric Acoustic neuroma: presents as unilateral deafness
olive, Tx rehydration and pyloromyotomy Narrow-angle glaucoma: presents as severe eye pain, due to Schwann cell proliferation on CN VIII, get MRI
headache, “seeing halos around lights”, eye feels as
Biliary atresia: presents as progressive jaundice in a hard as a rock; Tx iridotomy (give pilocarpine, aceta- Facial nerve tumor: presents as gradual-onset
1-2 month old due to lack of CBD; Dx HIDA scan + zolamide, or mannitol as first aid) unilateral facial paralysis (sudden-onset is Bell’s
phenobarbital (to stimulate GB contraction), Tx liver palsy), get MRI
transplant Orbital cellulitis: presents as cellulitis of eyelids w/
fixed/dilated pupil and limited eye ROM; get CT scan Parotid tumors: present as masses around angle of
Hirschsprung’s disease: presents as chronic then drain the pus mandible, two types – pleomorphic adenomas
constipation due to lack of nerves in distal colon, (benign, painless) and mucoepidermoid carcinomas
rectal exam can decompress bowel; Dx X-ray shows Chemical burns of eye: irrigate w/ plain water ASAP, (malignant, painful)
distended proximal colon, Tx surgical pull-through at hospital – irrigate w/ saline, remove corrosive
particles, and test pH before sending home; alkaline Parotid tumor management: get FNA or formal
Intussusception: presents as colicky abd pain that burns are worse than acid burns superficial parotidectomy (open biopsy contra-
lasts 1 minute then resolves, and currant jelly stools; indicated due to CN VII)
Dx/Tx barium or air enema (surgery if it doesn’t work) Retinal detachment: presents as flashes and floaters,
and a dark curtain being pulled down over the eye; Foreign bodies: any toddler w/ unilateral ENT sx
Child abuse: presents as retinal hemorrhages, SDH, Tx emergency laser “spot welding” (earache, rhinorrhea, wheezing) has a toy stuck in
multiple healed fx, and scalding burns; call child there; do endoscopy under anesthesia to remove
protective services Amaurosis fugax: emboli from carotid travels to
retina causing transient blindness; Dx fundoscopic Ludwig angina: abscess in floor of mouth due to
Meckel’s diverticulum: presents as LGIB in a child, exam reveals Hollenhorst plaque, a bright shiny spot tooth infection, Tx I&D and tracheostomy
Dx technetium uptake for ectopic gastric mucosa in a retinal artery
Surgery –Subspecialty Surgery

Bell’s palsy: sudden CN VII paralysis for no evident then resect (give mannitol, hyperventilate, and high- surgical emergency that requires immediate
reason, Tx antivirals immediately dose steroids while waiting) decompression (stent or perc nephrostomy) in
addition to IV antibiotics
CN VII trauma: sudden CN VII paralysis following Frontal lobe tumors: present as disinhibition,
facial trauma, will resolve spontaneously anosmia (CN I), ipsilateral blindness (CN II), and Lower UTI: presents as dysuria and cloudy, stinky
contralateral papilledema urine but no fever, chills, or flank pain; common in
Cavernous sinus thrombosis: presents as diplopia in young women, give antibiotics
a pt w/ sinusitis due to nerve (CN III/IV/VI) injury, Tx Craniopharyngioma: presents as GH deficit and
antibiotics, CT scan, and drainage of abscess bitemporal hemianopsia ◐◑ in a kid, Dx calcified Pyelonephritis: UTI that has ascended to the kidneys,
lesion in sella turcica presents with fever, chills, flank pain; Dx urologic
Epistaxis: nosebleed has different causes and work-up and Tx antibiotics
management depending on pt’s age Prolactinoma: presents as amenorrhea and
galactorrhea in a young woman, consider Prostatitis: presents as fever, chills, dysuria, back
Epistaxis × children: due to nosepicking, Tx local possibility of MEN1 syndrome, Tx bromocriptine pain, and tender prostate on rectal exam; give anti-
pressure and phenylephrine nasal spray biotics and don’t do any more rectal exams
Acromegaly: presents as huge hands, feet,
Epistaxis × teens: either cocaine abuse and septal tongue, and jaws in a tall man due to ↑GH; Dx get Urologic workup in kids: indicated for traumatic
perforation (requires packing) or juvenile somatomedin C levels and MRI, then resection hematuria and UTIs, since it may indicate congenital
nasopharyngeal angiofibroma (surgical resection) abnormalities otherwise undiagnosed
Nelson syndrome: presents as bitemporal
Epistaxis × elderly: due to HTN; requires packing, hemianopsia ◐◑ and hyperpigmentation in an Congenital malformations: posterior urethral valves,
BP control, and often surgical ligation of vessels adult due to pituitary microadenomas that grew hypospadias, epispadias, vesicoureteral reflex, low
to full size; Dx MRI then Tx surgical resection implantation of ureter, ureteropelvic obstruction
Vertigo: dizziness can be attributed to diseases of
inner ear or brain itself Pituitary apoplexy: presents with typical pituitary Posterior urethral valves: presents as anuria in a
adenoma sx, then sudden onset headache and newborn boy; cathereterize to empty bladder,
Vertigo × inner ear: “room is spinning”, Tx CNS sx due to bleeding into tumor; Dx MRI then then Dx voiding cystourethrogram, Tx resection
meclizine, phenergan, or diazepam emergent steroid replacement
Hypospadias: presents as urethral opening on
Vertigo × brain: “room is stable, but patient is Pinealoma: presents as loss of upper gaze and ventral side of penis, never circumcise since
spinning”, do a neurologic work-up “sunset eyes” due to compression of vertical prepuce is needed for surgical correction
gaze center in superior colliculi
––––––––––––––– NEUROSURGERY –––––––––––––– Epispadias: urethral opening on dorsal penis
Brain tumors in kids: present as cerebellar sx and
CNS diseases: vascular (sudden onset), metabolic headaches relieved by knee-chest position, due to VUR: presents as dysuria, fever, chills, and flank
(hours), infectious (days-weeks), tumors (months), posterior fossa lesions; Dx MRI, Tx resection pain in kids due to ascending UTI; give antibiotics
degenerative disease (years) and Dx IV pyelo and voiding cystogram to look for
Spinal cord tumors: present as back pain in reflux, long-term abx until child “grows out of it”
Vascular CNS diseases: TIA, ischemic stroke, someone who’s been treated for other cancers
hemorrhagic stroke, subarachnoid hemorrhage (spine is a common site of mets); Dx MRI, Tx neuro- Low implantation of ureter: asymptomatic in
surgical decompression boys, “wet with urine all the time” in girls b/c
TIA: brief neurologic deficit that fully resolves ureter drips into vagina instead of bladder; Dx
within 24 hours; due to thromboembolus from Neurogenic claudication: presents with pain on physical exam or IV pyelo, Tx surgical correction
internal carotid arteries exertion and relief with rest, but pain is position-
dependent and pulses are intact; Dx MRI, Tx neuro- Ureteropelvic obstruction: usually asymptomatic,
TIA management: carotid duplex, then aspirin surgical decompression but presents with colicky pain with large diuresis
and elective CEA (if >70% stenosis) (e.g. colicky pain following “beer drinking binge”)
Trigeminal neuralgia: severe facial pain lasting 60
Ischemic stroke: a neurologic deficit that doesn’t seconds; Dx MRI to r/o organic cause, then Tx anti- Hematuria: always get IV pyelo (CT scan if allergic or
resolve within 24 hours due to thromboembolus convulsants (radiofrequency ablation as backup) poor renal function) and cystoscopy to r/o cancers
from internal carotid arteries, Tx t-PA within 3
hours, otherwise observe and rehab Reflex sympathetic dystrophy: presents as severe Urologic cancers: renal cell carcinoma, transitional
pain months after a crush injury w/ sympathetic cell carcinoma, prostatic cancer, testicular cancer
Hemorrhagic stroke: severe headache and overload (cold, cyanotic, moist); Dx sympathetic
neurologic deficit that doesn’t resolve within 24 block, Tx surgical sympathectomy RCC: presents as hematuria, flank mass/pain, and
hours in uncontrolled HTN; Dx CT scan, Tx rehab paraneoplastic syndromes (PAPER – PTHrP, ACTH,
and control of HTN –––––––––––––––––– UROLOGY ––––––––––––––––– prolactin, EPO, renin); get CT scan if suspected,
Tx surgical resection
SAH: “worst headache of my life” due to rupture Testicular torsion: presents as severe testicular pain
of Charcot-Bouchard aneurysm in lenticulostriate and “high riding testicle with horizontal lie” due to Bladder TCC: presents as painless hematuria in
arteries; Dx CT scan to confirm SAH, then arterio- twisting of the cord; Tx emergent surgical detorsion, smokers; get IV pyelo and cystoscopy, then Tx
gram to locate aneurysm, then surgical clipping then orchiopexy (do not do any Dx tests) surgical resection, then lifelong f/u for high-rate
of local recurrence
CNS abscess: presents as space-occupying lesions Epididymitis: presents like testicular torsion but w/
but short-onset (weeks) and w/ nearby infections fever, pyuria, and cord is also tender; Dx U/S to rule Prostatic cancer: asymptomatic, detected as
(otitis media, mastoiditis); get CT scan then resect out torsion, then Tx antibiotics rock-hard nodule on rectal exam; get U/S-guided
needle biopsy, then TURP or radiation therapy
CNS tumors: presents as space-occupying lesions Obstructive UTI: presents as sepsis (fever, chills, (Tx flutamide and leuprolide if metastatic)
and progressive headache over months; get MRI flank pain) in someone passing a kidney stone;
Surgery –Subspecialty Surgery

Testicular cancer: presents as painless testicular


mass that doesn’t transilluminate; get radical
orchiectomy w/ biopsy, then f/u αFP or βHCG for
recurrence (Tx cisplatin or radiation if metastatic)

BPH: presents as dribbing, nocturia, and difficulty


voiding in an old man; put in Foley for 3 days, and Tx
α-blockers (tamsuosin), 5αR-blockers (finasteride)
+/- surgical prostate resection

Post-op urine retention: presents as a need to void,


but inability to do so; Tx straight cath at 6 hours
post-op and Foley after 2nd straight cath

Stress incontinence: presents as urine leaking w/


abd pressure due to weakened pelvic floor in a
multigravid woman; Tx surgical repair of pelvic floor

Kidney stones: presents as colicky flank pain and


hematuria +/- radiation to inner thigh, Dx plain X-rays;
Tx analgesics, fluids, and observation (<3 mm) or
shockwave lithotripsy (>3 mm)

Impotence: two types – organic and psychogenic,


management depends based on etiology

Psychogenic impotence: sudden onset w/o


traumatic cause, specific to partner or situation;
Tx psychotherapy

Organic impotence: sudden onset (trauma) or


gradual onset (atherosclerosis, diabetes); first-
line Tx is PDE5-blockers (sildenafil, Viagra)

–––––––––––– TRANSPLANT SURGERY –––––––––––

Transplant donors: almost everyone including brain-


dead pts, Hep and HIV pts to each other, metastatic
cancer pts for corneas

Transplant rejection: hyperacute (minutes), acute


(days/months), chronic (years)

Hyperacute rejection: preformed ab’s against donor


organ causes vascular thrombosis within minutes;
never seen since type-and-cross prevents this from
happening

Acute rejection: HLA mismatch causes macrophage


attack on donor tissue with lymphocytic infiltrate,
confirmed by biopsy

Liver rejection: indicated by elevated LFTs, first


step is to get U/S and Doppler to rule out biliary
obstruction and vascular thrombosis

Heart rejection: indicated by progressive heart


failure; management is getting routine ventricular
biopsies and Tx steroid bolus + OKT3

Chronic rejection: gradual deterioration of organ


function due to polymorphisms, irreversible and no
Tx available
Surgery – Orthopedic Surgery

–––––––––––– PEDIATRIC DISEASES –––––––––––– Anterior shoulder dislocation: presents w/ outward Open fx: require cleaning and reduction within 6
rotated arm + deltoid numbness due to axillary palsy hours to prevent ischemia of distal limb
Congenital hip dysplasia: presents as uneven gluteal
folds, posterior dislocating hips (with “snapping”) in Posterior shoulder dislocation: rare, presents w/ Gas gangrene: presents as tender, swollen,
a newborn; Dx U/S since hip isn’t calcified for X-ray, internally rotated arm held close to body discolored wound site w/ crepitus following a
Tx splinting penetrating injury; Tx high-dose penicillin G,
Colles fx: distal radial fx in osteoporotic women hyperbaric oxygen, and debridement
Avascular necrosis of femoral head: presents in a kid falling on outstretched hand; Tx reduction + cast
w/ limping, hip pain, and limited hip ROM; Dx X-ray,
Tx cast + crutches Monteggia fx: diaphyseal fx of proximal ulna + radial
dislocation, following direct trauma to ulna
Hip dislocation: presents as a chubby kid limping
due to groin pain, hip has limited internal rotation; Galeazzi fx: diaphyseal fx of distal radius + ulnar
Dx X-ray, Tx pin femoral head back in place dislocation, following direct trauma to radius

Septic hip: presents in toddlers who refuse to move Scaphoid fx: presents as wrist pain and tender
hip following a septic illness; Dx hip aspiration unter anatomic snuffbox in a young person falling on
anesthesia, Tx perc drain outstretched hand; Tx thumb spica cast

Osteomyelitis: presents in kids w/ severe localized Metacarpal neck fx: presents as swollen and tender
bone pain following a septic illness; Dx bone scan hand after punching a wall; Tx splint or plate
since X-ray will be false negative, Tx antibiotics
Hip fx: presents as shortened and externally rotated
Genu varum: aka bowlegs, <3 y/o observe since it’s hip in an osteoporotic woman; Tx depends on
normal, >3 y/o surgical correction location of fx

Genu valgus: aka knock-knee, observation okay Femoral neck fx: may lead to avascular necrosis of
femoral head; Tx prosthetic replacement of head
Osgood-Schlatter disease: presents in teens w/
persistent pain over tibial tubercle, aggravated by Intertrochanteric fx: less likely to cause avascular
quad contraction; Tx immobilization in cast necrosis; Tx ORIF

Club foot: presents as bilateral inverted feet in a Femoral shaft fx: Tx intramedullary rod fixation,
newborn; Tx serial plaster casts on adducted high risk of hypovolemic shock and fat embolisms
forefoot → hindfoot varus → equinus
Knee injury: all severe knee injuries present w/
Scoliosis: laterally curved spine, common in teenage swelling, and first step in management is an MRI
girls; Tx spine brace until mature, surgery if severe
Unhappy triad: injury to ACL, MCL, and medial
––––––––––––––– BONE TUMORS ––––––––––––––– meniscus following blow to lateral leg; Tx surgical
repair
Osteosarcoma: presents as low-grade knee pain in a
10-25 y/o, X-ray shows Codman’s triangle and ACL tear: presents as swelling, pain, and positive
sunburst appearance; consult a specialist anterior drawer sign; Dx MRI, Tx immobilization
and rehab
Ewing sarcoma: presents as low-grade epiphyseal
pain in a child, X-ray shows onion skinning, due to Meniscal tear: presents as pain, swelling, and
t(11;22); consult a specialist “clicking” of knee w/ forceful extension; Tx
arthroscopic repair to salvage meniscus
Metastatic bone cancers: usually from breast (lytic)
and prostate (blastic), causing bone pain and patho- Tibial stress fx: presents as localized tibial pain in a
logic fx; Dx bone scan, Tx chemoradiation soldier following a long march; Tx cast and f/u X-ray
in 2 weeks
Multiple myeloma: plasma B-cell tumor results in
CRAB – hyperCalcemia, Renal failure, Anemia, and Leg fx: common in pedestrian vs. car accidents; Tx
Bone pain (X-ray shows punched-out lesions); Dx cast for easily reduced fx, intramedullary nailing
urinary Bence-Jones protein, Tx chemotherapy otherwise, high risk of compartment syndrome

–––––––––––– ORTHOPEDIC TRAUMA –––––––––––– Achilles tendon rupture: presents as “popping noise”
in ankle following exertion w/ pain and swelling; Tx
Fractures: all require 2 X-rays (90° from each other) cast or surgery
including joints above and below fractured bone;
anticoagulation of choice is fondaparinux + leg Ankle fx: common after falling on inverted or
compression devices everted foot; Tx ORIF

Clavicular fx: typically between mid-to-distal third of ––––––––– ORTHOPEDIC EMERGENCIES –––––––––
clavicle; Tx figure-of-eight device for 4-6 weeks
Pain under cast: remove cast to examine

You might also like